SlideShare a Scribd company logo
1 of 41
1/41

                    Contents
01   Medicine                                             P. 1
02   Pediatrics                                           P. 16
03   Obstetrics and Gynecology                            P. 29
04   Surgery                                              P. 33
05   Orthopaedics and rehab                               P.39
06   Eye and ENT                                          P. 41
07   Psychiatry                                           P. 47
08   Forensic Med and Ethics                              P. 51
09   Miscellaneous                                        P. 53
        ขอบคุณเพื่อนๆ SI 116 ทุกคนที่ชวยกันพิมพขอสอบ
           ขอบคุณพี่ SI 115 ทุกคนที่ชวยกันจําขอสอบ
2/41
         01 Critical care                                                                                  1. HS                                   2. G6PD-def
         1. ชาย 50 ป หมดสติมา, no pulse, no respiration , ฟง heart sound ไมได แพทยใส tube            3. AIHA                                 4. Thalassemia with hemolytic crisis
         ,chest compression ติด EKG ไดดังรูป (เปนรูป V fib) ถาม Mx                                       5. PNH
                   1. IV Adenosine                       2. IV Epinephrine                        5. หญิงอายุ 18 ปเหนื่อยงายมา ? วัน มีซีดปานกลาง เหลืองเล็กนอย ตับไมโต มามโต 2 cm
                   3. chest compression ตอ              4. defibrillation                        CBC Hct 20 MCV 62 WBC 95,000 N85 L 15 Plt 250,000 reti 8 ถาม Dx
         2. หญิง 65 ป ใจสั่น หมดสติไป 5 นาที ญาตินําสงรพ. รูสติดี .. HR 40 /min EKG ดังรูป              1. Thalassemia                          2. Iron Def
         จงใหการวินิจฉัย                                                                                  3. G6PD                                 4. Hereditary spherocytosis
                                                                                                           5. AIHA
                                                                                                  6. ชายอายุ 50 ป เปน cirrhosis มาดวยอาเจียนเปนเลือด
                                                                                                           PE : shifting dullness positive, no hepatomegaly, splenomegaly 4 cm
                                                                                                           CBC : WBC 10,000 , Plt 60,000 , %N 32 , %L 76
                                                                                                  จะใหสวนประกอบของเลือดอะไร
                                                                                                           1. PRC                                  2. Cryoprecipitate
                 1. 3rd heart block                                                                        3. Ciyo-less                            4. Platelet conc.
                 2. 2nd degree AV block mobitz I                                                           5. FFP
                 3. 2nd degree AV block mobitz II
                 4. 1st degree block                                                              01 Neurology
         3. ชายไทยอายุ 23 ป ถูกผึ้งตอยมาดวย wheel 3+, อาเจียน BP 80/50 mmHg PE:edema           7. ผูปวยหญิงมาดวย proximal muscle weakness เปนมากตอนบาย ๆ มี diplopia, หนังตา
         รอบปาก Management?                                                                       ตก, DTR 2+ สงตรวจอะไรเพื่อการวินิจฉัย
                 1. H2 antagonist                       2. H1 antagonist                                     1. LP                                2. Prostigmine test
                 3. Dexamethasone IV                    4. Adrenaline IM                                     3. Electrolyte                       4. Muscle biopsy
                                                                                                  8. เด็กอายุ 8 เดือน แมพามาดวยอาการไขสูง, anterior fontanel โปง, Brudzinski positive
         01 Hemato                                                                                จะทําอยางไร
         4. ผูปวยหญิงอายุ 30 ป เหนื่อยเพลียมา 2 สัปดาห ปสสาวะเขมขึ้น ไมมีไข no                       1. ขอความยินยอมพอแมเพื่อ LP
         hepatosplenomegaly, moderate pale, mild icteric sclera, WBC 3,600, Plt 234,000 , blood              2. LP เลยเพราะเรงดวน
         smear ดังรูป จงใหการวินิจฉัย                                                                       3. CT brain


National Test - Medicine                                                         หนาที่ 1              National Test - Medicine                                                            หนาที่ 2
3/41
                     4. ใหยากันชัก, ATB หลัง H/C                                                    13. หญิง อายุ 65 ป เปนเบาหวานมา 5 ป เมื่อ 2 วันกอนปสสาวะแสบขัด วันนี้มีไขสูง
                     5. สงตอ                                                                       หนาวสั่น ซึมลง, T 39 C BP 140/80 P 120, U/A protein trace, sugar neg, SpGr 1.018,
          8. หญิง 27 ป มีอาการปวดศีรษะตุบๆ ปวดขางเดียว เปนสลับขางกันเปนๆหายๆ เดือนละ           bacteria numerous, RBC 10-15, WBC 50-60, no cast/ept cell จงใหการรักษา
          4-5 ครั้ง ขณะนี้ on oral contraceptive มานาน … ป ตรวจรางกายไมพบ neurological                      1. Ampicillin                           2. Norfloxacin
          deficit ,…….                                                                                         3. Gentamycin                           4. Ceftriaxone
                     1. CT brain                              2. MRI brain                                     5. Cotrimoxazole
                     3.ให diazepam                           4.ให tramadol                         14. ชาย 33 ป ปวดศีรษะเรื้อรังมาหลายเดือน ใจสั่น เหนื่อยงาย หนาแดง sweating ปวดหัว
                     5.ให amitriptyline                                                             มากขึ้น BP 180/110, P 110/min flushing การตรวจเพิ่มเติมเพื่อการวินิจฉัยโรค
          10. ผูปวยชาย เดินชา มี resting tremor Rt > Lt มี Cogwheel rigidity ใหการรักษาอยางไร             1. Urine VMA
                     1. Selegiline                            2. Levodopa                                     2. CT brain
                     3. Benhexole                             4. Bromocriptine                                3. Serum electrolyte
          11. ชายอายุ 24 ป มีอาการชักเกร็งกระตุกทั่วตัว ตาเหลือก ไมรูสึกตัวมานาน 1 ชั่วโมง มี
                                                                                                              4. TFT
          ประวัติโรคลมชักตั้งแตอายุ15ป ชักปละ 3-4 ครั้ง ขาดการรักษามา 1ป ขณะตรวจไมมี
                                                                                                              5. IVP
          อาการชักแลว การรักษาที่เหมาะสมที่สุดในผูปวยรายนี้
                     1. Phenytoin                             2. Diazepam                            15. ชาย 25 ป ปวดศีรษะมานาน 1 mo วันนี้ออนเพลีย แขนขาไมมีแรง BP 190/120 motor
                     3. Midazolam                             4. Phenobarbital                       power 3/5, P 110 investigate
                     5. Gabapentin                                                                            1. Urine VMA <<<<เฉลย
                                                                                                              2. CT brain
          01 Nephro                                                                                           3. Serum electrolyte
          12. ผูปวยหญิงไทยอายุ 43 ป แขนขาออนแรงมา 1-3 วัน 1 เดือนกอนมีอาการขาบวม
                                                                                                              4. TFT
          ไปพบแพทยที่คลินิกได hydrochlorothiazide (50 mg) วันละเม็ด อาการดังกลาวเกิดจาก
                                                                                                              5. IVP
          อะไร
                    1. Hypokalemia                     2. Hyponatremia                               16. ชายอายุ 39 ป มารพ.ดวย ปสสาวะสีน้ําลางเนื้อมา 2 วัน ตรวจรางกายพบ puffy eyelids,
                    3. Hyperglycemia                   4. Hypocalcemia                               BP 140/90, UA red brown color urine, Sp.Gr. 1.025 Albumin 1+ RBC 100/HF, WBC
                    5. Hypomagnesemia                                                                10/HF with rbc cast รักษาอยางไร
                                                                                                              1. Norfloxacin                        2. Ceftriaxone

National Test - Medicine                                                             หนาที่ 3           National Test - Medicine                                                              หนาที่ 4
4/41
                  3. Furosemide                            4. Prednisolone                                    5. Congenital heart disease
       17. ผูปวยหญิงอายุ _ ป สูบบุหรี่ 30 pack year อาการเขาไดกับ CA Lung, no underlying      21. หญิงอายุ 45 ป มีไขตอนเย็นๆ ไอมีเสมหะ ไมมีเลือดมา 3 wk ได amoxiclav 4 days
       disease                                                                                     levofloxacin 5 days อาการไมดีขึ้น CXR alveolar infiltration with RUL cavity ถามวา Mx
       ตรวจ Lab : E’lyte : Na 125, K _ , Cl 97 ; Urine Na 10, Urine osmol 50                       ตอไปคือ
       อะไรคือสาเหตุของhyponatremia ในผูปวยรายนี้                                                           1. Sputum AFB                         2. Bronchoscope
                  1. SIADH                                 2. Low salt intake                                 3. CT chest
                  3. Renal wasting                         4. Thiazide                                        4. FNA
                  5. Primary polydipsia                                                            22. คนงานโรงงานกระเบื้อง... ตอมาสงสัยวาเปนโรคมะเร็งปอด ในการวินิจฉัยวา เกิดโรค
       18. ผูปวย heart failure มี fine crepitation ปสสาวะไมออก ให furosemide iv 2 doses ยัง   จากการประกอบอาชีพ ตองทําอะไรเปนอันดับแรก
       ปสสาวะไมออกอีก BUN 75,Cr 5.2,CO 12, Na 131,K 6 ควรทําอะไรตอ                                         1. ตรวจสอบขอมูลเกี่ยวกับการทํางานอยางละเอียด
                  1. Dialysis                              2. IV furosemide                                   2. วินิจฉัยใหไดวาเปนมะเร็งปอดจริง
                  3. Ca gluconate                          4. Restrict salt diet                              3. หาขอมูลวามีหลักฐานสัมพันธกับมะเร็งปอด
                  5. NaHCO3                                                                                   4. ตรวจวัดปริมาณสารวาพอจะทําใหเกิดมะเร็งปอดหรือไม
       19. ผูปวยหญิงอายุ 43 ป มีอาการออนเพลีย แขนและขาออนแรงมา 2 วัน เมื่อประมาณ 1                       5. –
       เดือนกอนมีอาการขาบวมเล็กนอย ไดรับยา HTCZ (50mg) 1 เม็ดตอวัน รับประทานทุกวัน             23. ผูปวยชายอายุ 50 ป สูบบุหรี่ 2-3 มวน/วัน มานาน 30 ป มีอาการแนนหนาอก เหนื่อย
       อาการผูปวยเกิดจากเหตุผลใด                                                                 มา 2 วัน หนาและแขนบวม ตรวจพบ not pale, face & upper extremity edema, superficial
                  1. Hypo K                                2. Hypo Na                              vein dilatation at chest wall ทํา CXR ผลปกติ สาเหตุที่นาจะเปนไปไดในผูปวยรายนี้
                  3. Hyperglycemia                         4. Hypo Ca                                         1. Cor pulmonale                      2. Cardiac temponade
                  5. Hypo Mg                                                                                  3. Pulmonary tuberculosis             4. Bronchogenic carcinoma
                                                                                                   24. ชาย 25 ป 2 สัปดาหกอนดื่มสุรา เมาและอาเจียนมาก และหมดสติ ตรวจพบ pneumonia
       01 Res                                                                                      ที่ superior segment of right upper lobe ตรวจเสมหะพบ pleomorphic gram positive cocci
       20. ผูหญิงอายุ 20 ป มีไอเรื้อรัง ตั้งแต 10 ปกอน บางครั้งมีเสมหะเขียว ตอนเด็กๆเคยมี     vary in size and shape ใหการรักษาดวย co-amoxyclav นาน 3 วัน ยังคงมีไข ตรวจ x-ray
       Respiratory tract infection เยินๆตอนอายุ 7 ป CXR : peribroncheal thickening                เพิ่มเติมพบมี pleural effusion เจาะน้ํามาตรวจได น้ําสีเหลืองขุน WBC 2,500 (N 95%) ,
       จงใหการวินิจฉัย                                                                            glucose 30 mg/dl , LDH 900 ตรวจ gram stain พบเชื้อแบบเดิม จงบอกการปฏิบัติที่
                  1. Lung abscess                            2. Bronchiectasis                     เหมาะสมในผูปวยรายนี้
                  3. Pulmonary tuberculosis                  4. Chronic bronchitis                            1. เพิ่มยา aminoglycosides


National Test - Medicine                                                       หนาที่ 5                  National Test - Medicine                                                    หนาที่ 6
5/41
                 2. เปลี่ยนเปนยากลุม carbapenem                                                   01 Endocrine
                 3. ใหยาเดิม + CT chest                                                            29. ผูปวยหญิงอายุ 28 ป มีไข ออนเพลีย มา 1 สัปดาห เหนื่อยงาย ใจสั่น PE : T 38 °C, P
                 4. ใหยาเดิม + ใส ICD                                                             120/min, tremor both hand Neck : marked tender thyroid gland and diffuse
                 5. ใหยาเดิม + เจาะน้ําในอีก 24 hrs.                                               enlargement จงใหการวินิจฉัย
                                                                                                               1. Acute thyroiditis                   2. Grave’s disease
        01 Toxico                                                                                              3. Hashimoto’s thyroiditis             4. Suppurative thyroiditis
        25. ผูปวยชาย กินยาฆาตัวตายมา มีอาการ sweating CVS: normal , RS: diffuse lung                        5. Papillary CA thyroid
        infiltration , muscle มี fasciculation                                                      30. ผูปวยหญิง อายุ 28 ป มีอาการเหนื่อยมา 1สัปดาห P120/min RR20/min BP120/80
                   1. Salicylate                           2. Paracetamol                           PE : mildly diffuse thyroid enlargement, markedly tenderness จงใหการ Dx
                   3. Organophosphate                                                                          1. Supperative thyroiditis
        26. ชายอายุ 18 ป ภูมิลําเนา จ.พิจิตร 1….กอน กินซุปหนอไมกับเพื่อนอีก 3 คน ตอมาเริ่มมี              2. Acute thyroiditis
        อาการ weak แขนขาออนแรง BP↓ ผูปวยมารับการ intubation และใสเครื่องหายใจแลว                          3. Hashimoto’s thyroiditis
        อาการดีขึ้น เพื่อนอีก 2 คนเสียชีวิตแลว คิดวาเกิดจากพิษใด                                             4. Papillary cell CA
                   1. Botulinum toxin
        27. ผูปวยชายไทย ภูมิลําเนาสังขละ กาญจนบุรี มีรพ.ดวยอาการ proximal muscle                 01Cardio
        weakness with hyperpigment skin, hyperkeratotic patch at palm and sole, white line nail,    31. ผูปวยมี pulse 90 irregular, PE : diastolic rumbling murmur, opening snap ขอใดถูก
        decreased sensation                                                                                    1. มีโอกาสเกิด.....เทาคนปกติ
                   1. Penicillamine                                                                            2. มีโอกาสเกิด embolic stroke มากกวาคนทั่วไป 15-20 %
                   2. Defuroxamine                                                                             3. มีโอกาสเกิด pulmonary hypertension เพราะมีลิ้นหัวใจรั่ว
                   3. Calcium EDTA                                                                  32. หญิงไทยอายุ 42 ป กอนหนานี้แข็งแรงดี มีไขมา 1 wk. หอบเหนื่อยมา 3 วัน ไอ นอน
                   4. Hemodialysis                                                                  ราบไมได T 38 C , BP 100/70 mmHg , P 100/min , RR 32/min , subconjunctival
        28. ผูปวยหญิงอายุ 25 ป กินน้ํายาลางหองน้ํามา 1 ชั่วโมงกอนมาโรงพยาบาล ขอใดเปนผล      hemorrhage , LV heaving (systolic murmur grade 3 at apex) มีตุมดําแดงกดเจ็บที่ปลายนิ้ว
        ระยะยาวที่อาจเกิดขึ้น                                                                       ถาม Dx
                   1. Esophagitis                          2. ……. pneumonitis                                  1. Atrial myxoma
                   3. Mediastinitis                        4. Esophageal stricture                             2. Acute rheumatoid carditis
                                                                                                               3. Bacterial endocarditis


National Test - Medicine                                                          หนาที่ 7                National Test - Medicine                                                             หนาที่ 8
6/41
                  4. Ruptured chordae tendinae                                                  37. ผูปวยกิน amoxicillin 10 วัน อาการเจ็บคอลดลง ตอมา มีทองเสียครั้งละ 1 แกว มีมูก
       33. ผูปวยไมรูสึกตัว คลําชีพจรไมได EKG : severe bradycardia with intraventricular   ปนเล็กนอย 3-4 ครั้งตอวัน นาน 7 วัน เกิดจากเชื้อใด
       conduction delay                                                                                    1. E.histolytica                       2. E.coli
                  1. Atropine                                                                              3. Campylobactor jejuni                4. Shigella
                  2. Epinephrine                                                                           5. C.difficile
                  3. External pacing                                                            38. ชาย 50 ป HT มานาน ปวดหลัง 1 day PTA, PE: P 110/min, BP 90/60 mmHg,
       34 หญิง 55 ป CRF 2 ป, DM 2 ป, HT 4ป กินยาสม่ําเสมอ PE: BP 130/80 ตรวจตาพบ            abdominal mass 8 cm with bruit. Investigation?
       proliferative diabetic retinopathy Lab:Cr 2 ถามปริมาณโปรตีนที่ควรกินใน 1 วัน                        1. Angiogram                           2. CT abdomen
                  1. 10 g                                 2. 20 g                                          3. MRI Abdomen                         4. Acute abdomen plain film series
                  3. 30 g                                 4. 40 g                                          5. U/S whole abdomen
                  5. 50 g                                                                       39. ชาย 30 yrs, diarrhea 10 times in 8 hrs
       35. เด็กหญิง 10 ขวบ เปนโรคผนังกั้นหัวใจรั่วแตกําเนิด รักษาเรื่องเหนื่อยจากหัวใจ        PE : P 110/min, BP 90/60 , flat neck vein, dry lips
       ลมเหลวหลายครั้ง หลัง 3 ขวบ อาการดีขึ้น ครั้งนี้เหนื่อยมากมา 2 เดือน แตนอนราบได        ให สารน้ําอะไร
       PE : central cyanosis, clubbing of fingers and toes, loud P2, diastolic blowing murmur              1. ORS                                 2. 5%D/W
       grade 3/6 at left upper sterna border, lungs-clear ครั้งนี้เหนื่อยจากอะไร                           3. 5%D/N/2                             4. NSS/2
                  1. Aortic regurgitation                 2. Pulmonary embolism                            5. NSS
                  3. Eisenmenger syndrome                                                       40. ชายอายุ 41 ป แข็งแรงดีมาตลอด ไมมีประวัติโรคมะเร็งในครอบครัวเห็นเพื่อนรวมงาน
       36. หญิงหมดสติ ให EKG มา                                                                2 คน อายุ 41 และ 45 ปเปนมะเร็งลําไส จึงตองการตรวจเช็ค วิธีใดเหมาะสมที่สุด
                  1. Sinus arrest                                                                          1. Barium enema
                  2. Sinus bradycardia                                                                     2. Carcinoembryogenic antigen
                  3. Complete heart block                                                                  3. Colonoscopy
                  4. Left bundle branch block                                                              4. Stool occult blood
                  5. Premature atrial beat                                                                 5. ไมตองทําการตรวจ screening
                                                                                                41. หญิง 22 ป ไมเคยเหลือง ตรวจเลือดกอนเขาทํางานพยาบาล พบ HBsAg +ve , Anti-
       01 GI                                                                                    HBc IgM neg , LFT normal ทําอยางไร
                                                                                                           1. ให HBV vaccine


National Test - Medicine                                                         หนาที่ 9             National Test - Medicine                                                     หนาที่ 10
7/41
                  2. ให interferon alpha                                                                    2. Artesunate+tetracycline
                  3. ตรวจ AFP                                                                                3. Chloroquine+….
                  4. U/S liver                                                                               4. Artesunate+mefloquine
                  5. ตรวจ HBs Ag ซ้ําอีก 6 เดือน                                                             5. Primaquine+….
       42. ผูชายกินเหลามานาน ปวดทอง อาเจียน admit NPO แลวอาละวาด ดึงสายน้ําเกลือ เปน         47 ชาย อายุ 25 ป มาดวยเรื่องปวดศีรษะ มีไข
       อะไร                                                                                       PE :stiff neck- negative,CT brain-normal
                  1. Hypoglycemia                                                                 CSF : protein 65 ,glucose 35,wbc??, L 95%,gram stain and india ink-negative
                  2. Hepatic encephalopathy                                                                  1. Acyclovir                          2. Ceftriaxone
                  3. Wernicke encephalopathy                                                                 3. Cotrimoxazole                      5. Amphotericin B
                  4. Delirium tremens                                                             48. ผูปวยชายอายุ 20 ป มาดวยไขสูง หนาวสั่น ( T 38 c ) CBC: ปกติ plt100,000/ul ให
                  5. Alcohol intoxication                                                         Blood smear ดังรูป จงให Diagnosis
       43. ผูปวยอายุ 29 ป ทองผูกบอยๆ stool exam ปกติ ทําอยางไร                                         1. PF                                 2. PV
                  1. GI bulk forming                                                                         3. PO                                 4. PM
       44. ผูปวยชาย อายุ 40 ป มีกอนโตที่บริเวณคอขวามา 4 สัปดาห เดิมแข็งแรงดี ขนาดเสน                   5. PV+PF
       ผานศูนยกลางกอน 4 ซม., firm, not fixed and no tenderness ทําอะไรตอ                      49. ผูปวยชายไทยมีผื่นที่ขาหนีบ 2 ขาง ลักษณะ Annular with scale, คัน สงตรวจ KOH จะ
                  1. ENT exam                          2. Tuberculin skin test                    พบลักษณะใด
                  3. CXR                               4. Lymph node Bx                                      1. Branching septate hyphae
                                                                                                             2. Short hyphae with yeast
       01 ID                                                                                                 3. Budding yeast with thick wall
       45. ผูปวย ชายไทย อายุ 30 ป มีไข 10 วัน ใหรูป blood smear มา รูปมี infected RBC ใหญ   50. ผูปวยกอนหนานี้มีไข กินยาลดไขแลวไขหายไปแลว ตอมามี erythematous nodule ที
       เปนอะไร                                                                                   ขา 2 ขาง การสงตรวจขอใด มีประโยชนในการ diagnosis
                  1. P. Ovale                             2. P. vivax                                        1. CXR
                  3. P. Falciparum                        4. P. Malariae                                     2. U/A
                  5. Mix                                                                                     3. CBC
       46. ผูปวยชายอายุ 30 ป มีไข ตรวจพบ double chromatin ring form ใหยาอะไร                 51.ไขพยาธิดังภาพ ใหยาอะไร
                  1. Chloroquine                                                                             1. Albendazole

National Test - Medicine                                                             หนาที่ 11          National Test - Medicine                                                         หนาที่ 12
8/41
                  2. Mebendzole                                                                          1. rickettsial infection
                  3. Metronidazole                                                                       2. chikungunya
                  4. Praziquantel                                                                        3. rheumatic heart disease
       52. หญิงมาดวยอาการอาเจียนเปนเลือดสีดํา 2 ครั้ง ประวัติ Dengue ใหผล labมา ถามวาผล
       แทรกซอนของโรค                                                                           01 Dermato
                  1. Bleeding time                                                              57. ชาย 70 ป มีผื่นนูนแดงที่ขา คัน ชอบเปนตอนปใหม Dx
                  2. Euglobulin test                                                                     1. Static eczema
       53. ผูปวยหญิงอยูกาฬสินธุ มีอาการเหนื่อย มีไข ไอ มา 2 สัปดาห ตรวจรางกาย พบ T                2. Xerotic eczema
       39°C, BP 140/60 mmHg, RR 28/min, P 100/min, Hepatomegaly 2 FB below Rt. Costal                    3. Chemical eczema
       margin, Splenic dullness positive, CXR มี Pulmonary infiltration ที่ Lower lung field,            4. Irritant contact dermatitis
       ultrasound มี multiple hypoechoic lesions ที่มาม, BUN 40 mg/dl, Cr 5 mg/dl, FBS 260              5. Allergic contact dermatitis
       mg/dl จงใหการวินิจฉัย
                  1. Mellioidosis                         2. Salmonella                         01 Nutrition
                  3. Amoebic Liver Abscess                4. Leptospirosis                      58. คนไขตับโต LFT ผิดปกติเล็กนอย (AST 46 U/ml, ALT 60 U/ml), U/S พบ
       54. ผูปวยหญิงเปน SLE กิน prednisolone 60 mg มา 3 เดือน ไอเล็กนอย ได sputum มาเปน   hepatomegaly เกิดจากการขาดอะไร
       ตัวอะไร                                                                                            1. Zinc                               2. Tretinol
                  1. Ancylostoma                          2. Enterobius vermicularis                      3. Selenium
                  3. Strongyloides stercolaris            4. Paragonimus                        59. ผูชายกินเหลามาก มี sign of CHF เปน cardiac beri beri ถาม managemen
       55. ผูปวยชายอายุ 28 ป มีไขสูงปวดศีรษะ Stiffneck +ve มีประวัติเลนน้ํา นึกถึง                   1.Thiamine                            2. Diazepam
                  1. N.fowleri                            2. N.meningitis                                 3. glucose
                  3. P.aerugenosa                         4. Hemophilus spp.                    60. ชาย 70 ป Alzheimer กินไดนอย ไมสําลัก ตองหยอดอาหาร ทําไงดี
                  5. S.aureus                                                                             1. NG                                 2. Nasoduodenal
       56. ผูปวยชาย ภูมิลําเนา จ.นราธิวาส มีไข และปวดเขา 2 ขาง ตอมามีปวดขอเทาและ                  3. Nasojujunal                        4. PEG
       ขอศอกเพิ่ม ตรวจรางกายพบวามีไข, ฟง heart no murmur, มี erythematous maculopapular              5. PEJ
       rash
       CBC มี N 40%, L 50%, atypical lymphocyte 5%, platelet ปกติ จงใหการวินิจฉัย              01 Ambu

National Test - Medicine                                                    หนาที่ 13                 National Test - Medicine                                             หนาที่ 14
9/41
       61. ผูปวยหญิง อายุ 40 ป, สูง 150 cm, น้ําหนัก 70 kg, BP 150/80 mmHg, ชอบกินอาหาร   02 Develop
       รสจัด จงใหการรักษา                                                                   1. เด็กซน 8 ขวบ ป.1 ซนมาก ลืมสงงาน งานหายบอย มาพบทาน....
                  1. Furosemide                           2. ACEI                                        1. เปน MR                              2. พาไปพบจิตแพทย
                  3. B-blocker                            4. CCB                                         3. แนะนําใหปรึกษาคุณครูที่ ร.ร.        4. เปนปกติของเด็กวัยนี้
                  5. ควบคุมอาหาร + ลดน้ําหนัก                                                2. เด็ก 9 เดือน เกาะยืนได ยังไมเดิน พูดเปนคําไมมีความหมาย ยอมใหทุกคนอุม ฟนยังไม
                                                                                             ขึ้น ไมยอมใหของเมื่อขอ อะไรผิดปกติ
                                                                                                         1. walking                              2. speech
                                                                                                         3. stranger anxiety                     4. tooth eruption
                                                                                                         5. Give toys on request
                                                                                             3. เด็ก 2ป มาปรึกษาปญหาสุขภาพ ขณะนี้กินอาหาร 3 มื้อ ไขวันละ 1ฟอง นม4กลองตอ
                                                                                             วัน อาหารวางเปนผลไม นน. 16kg สูง 90 cm จงใหคําแนะนํา
                                                                                                         1. เพิ่มอาหารมื้อกอนนอน
                                                                                                         2. เพิ่มไขมื้อละ 1ฟอง
                                                                                                         3. ลดอาหารเปนวันละ 2มื้อ
                                                                                                         4. ลดนมเหลือวันละ 2กลอง ตอวัน
                                                                                                         5. เพิ่มอาหารวางเปนน้ําผลไมอีก 2มื้อ
                                                                                             4. เด็ก 2ป น้ําหนัก 12kg สูง 90 cm กินขาว 3มื้อ ผลไม นม 8 0z x 3, พูดได 2 คํา, วิ่งได, ได
                                                                                             vaccine HBV x 3, OPV x 3, DPT x 3 แนะนําอยางไร
                                                                                                         1. เพิ่มนม                              2. เพิ่มมื้ออาหาร
                                                                                                         3. รับ OPV, DPT                         4. ปรึกษากุมารแพทยดานพัฒนาการ
                                                                                             5. ผูปวยเด็กอายุ 6 เดือน ยังชันคอไมได แขนขาขยับไดปกติดี ไมมีไข ไมซึม ผลการตรวจ
                                                                                             รางกาย afebrile,R 30/min, BP 80/50 mmHg, P 100/min, Head circumference 46 cm,
                                                                                             anterior fontanelle 3x3 cm, Increase muscle tone, DTR 3+, Babinski’s – dorsiflexion
                                                                                             both sides ควรตรวจอะไรเพิ่มเติมเพื่อใหไดการวินิจฉัย
                                                                                                         1. TORCH titer                          2. Chromosome study
                                                                                                         3. Thyroid function test                4. Bone scan


National Test - Medicine                                                      หนาที่ 15             National Test - Pediatrics                                                          หนาที่ 16
10/41
                     5. U/S brain                                                                  CXR : haziness
           6. ผูปกครองพาเด็ก 6ขวบ มาขอใบยกเวนไมตองเขาโรงเรียน ผลการทดสอบสติปญญา              จะสง Lab อะไร
           IQ=60 ควรทําอยางไร                                                                               1. Microplasma titer                    2. ICD
                     1. ออกใบรับรองไมตองเขาโรงเรียนให                                                    3. Nasopharygeal swab
                     2. ออกใบรับรองเขาเรียนไดโดยมีอาจารยพิเศษ                                   11. เด็กชาย 6 ป ไข 1 วัน เหนื่อยหอบ หายใจลําบาก PE T 40 ° , RR 33 /min, Chest :
                     3. ออกใบรับรองเสื่อการเขาเรียนเปน 9ป                                       inspiratory stridor with Suprasternal retraction, inspiratory sonorous rhonchi การตรวจ
                     4. ไมออกใบรับรอง เพราเด็กสามารถเรียนได                                      ในขอใดชวยวินิจฉัยมากที่สุด
                     5. ใหเจาหนาที่ตรวจสภาพแวดลอมบานกอนออกใบรับรอง                                     1.U/S chest                             2. CT chest
           7. เด็กอายุ 1ป แมสังเกตเห็นวาตาขวาเหลเขาในมา5เดือน พัฒนการปกติ ถาไมรักษาจะ                 3. CXR PA & Lat film                    4. CXR inspiration & expiration film
                     1. Nystagmus                          2. Glaucoma                                       5. Lat soft tissue technique film
                     3. Amblyapia                          4. Visual field defect                  12. เด็ก 10 ป ไอ แนนหนาอก หอบ 5 วัน face, chest & upper extremities edema , liver 2
                     5. Limit ocular movement                                                      cm. Below Rt.Costal margin. What is the diagnosis?
                                                                                                             1. Croup                                2. Angioedema
           02 Respi                                                                                          3. SVC obstruction                      4. CHF
           8. ผูปวยเด็กชาย อายุ 6 เดือน มีไขและไอมากขึ้นหลังจากเปนหวัดมา 2 วัน, T 38°C, R                5. Constrictive pericarditis
           44/min, Inspiratory
           stridor, retract intercostals, nasal flaring, occasionally rhonchi จงใหการวินิจฉัย     02 nephro
                       1.Viral croup                            2.Bronchiolitis                    13. เด็ก 5 ป ไข 2 วัน ปสสาวะสีเขม เหนื่อยเพลียมากขึ้น, PE mildly pale, CBC Hb 6 Hct
                       3.Whooping cough                         4.Acute epiglottitis               ... WBC normal (N85 L15) Platelet 14,300, U/A dark brown urine, urobilinogen +ve,
                       5.Asthmatic bronchitis                                                      Complication?
           9. เด็ก 8 เดือน มีไข ซึม ไมยอมดูดนม น้ําลายยืด มา 2 วัน PE : T 39 c, RR 30 /min,                 1.Gallstone                            2.Acute renal failure
           drooling, noisy breathing, harsh breath sound, bulging posterior pharynx                           3.Pulmonary embolism
                       1. viral croup                           2. bacterial tracheitis            14. เด็กหญิง 14ป บวมทั่วตัว ขาบวม
                       3. retropharyngeal abscess               4. acute epiglottitis              PE : T37.8 *C,BP140/90 mmHg, mild anemia
           10. เด็กหญิงอายุ 8ป มีไข ไอ หอบ มา 3วัน กอนหนานี้ 4วัน มีไขต่ําๆ                              pitting edema both legs, oral ulcer, MP rash at palms and soles
           PE : Decrease breath sound at Rt. , Trachea shift to left                               Lab : CBC Hb 8 g/dl WBC 4,000 thrombocytopenia


National Test - Pediatrics                                                            หนาที่ 17       National Test - Pediatrics                                                            หนาที่ 18
11/41
                      U/A protein 3+ WBC 30-40 cell RBC cast                                              19. เด็กทารกอายุ 3 วัน คลอดปกติ กินนมแมไดดี BW 3200 g ปกติดีทุกอยาง ตรวจพบวา
           การตรวจทางหองปฏิบัติการใดที่ชวยวินิจฉัย                                                      เหลือง TB 12 mg/dL Hct 50% ทําอยางไร
                      1.ASO titer                               2.C3                                                 1. ใหดูดน้ําเพิ่ม
                      3....                                     4.ANA                                                2. หยุดนมแม 1-2 วัน เพราะเกิดจากการกินนมมารดา
                      5.albumin,cholesterol                                                                          3. ตากแดดออนๆ
           15. ผูปวยเด็ก อายุ 6 ป อาเจียนมาก ตรวจ Electrolyte ผลที่ไดจะเปนอยางไร                               4. กลับบานไมไดใหobserve
                      1,Hypochloremia                           2.Hypokalemia                                        5. admit on photo
                      3.Metabolic alkalosis                                                               20. ผูปวยเด็ก preterm BW = 2,200 g มารดาคลอด C/S due to abruptio placenta apgar
           16. เด็กชาย อายุ 6 สัปดาหแมพามาเพราะอาเจียนมาก serum E’lyte เขาไดกับ                       score 5, 8
                      1. hyperchloremic hyponatremic metabolic acidosis                                   หลังคลอด 12 ชม. เหลือง ทํา exchange สาเหตุสําคัญที่ตองตรวจการไดยินของเด็กคนนี้
                      2. hyperchloremic hypokalemic metabolic acidosis                                    คือ
                      3. hyperchloremic hyperkalemic metabolic acidosis                                              1. BW = 2,200 g                       2. abruptio placenta
                      4. hypochloremic hypokalemic metabolic alkalosis                                               3. apgar score 5, 8                   4. preterm
                      5. hypochloremic hyponatremic metabolic alkalosis                                              5.เหลือง
           17. เด็ก 8 ป bilateral vesico ureteral reflux มีติดเชื้อซ้ําบอยๆ BUN/Cr ขึ้นเรื่อยๆ ลาสุด   21. ทารกอายุครรภ 44 สัปดาห น้ําหนักแรกคลอด 3400 g มี meconium stained
           BUN 55, Cr 3.8, Na 135, K 4.3, HCO3 16, ปสสาวะตอวัน 900 ความดันปกติ จงใหการ                 amniotic fluid คลอดทางชองคลอด APGAR score 2,5 ที่ 1 และ 5 นาที ตามลําดับ
           รักษา                                                                                          หลังคลอด 10 ชม. มีชักเกร็งกระตุกทั่วตัว ผลตรวจเลือด blood glucose 50 mg/dl Hct
                      1. salt diet ลดลง                         2. K diet ลดลง                            58% สาเหตุการชักในทารกรายนี้จากความผิดปกติใด
                      3. oral NaHCO3                            4. phosphate diet                                    1. meningitis                         2. hypoglycemia
                      5. dialysis                                                                                    3. polycythemia                       4. perinatal asphyxia
                                                                                                                     5. intraventricular hemorrhage
           02 Neonate                                                                                     22. เด็กคลอดกอนกําหนด แมเปน DM หายใจหอบเหนื่อย เร็ว CXR พบ hyperaeration
           18. เด็กแรกเกิดเหลืองหลังคลอด 3-4 wk กินนมแม ไมโต ลิ้นจุกปาก ไมดูดนม ทองอืด จะ             with generalized haziness with air bonchogram, Dx?
           ตรวจอะไรเพื่อ Dx                                                                                          1. fluid                              2. MAS
                     1. Chromosome study              2. TFT                                                         3. RDS                                4. bronchopulmonary dysplasia
                     3. GH level                      4. GI follow through                                           5. transient tachypnea of the newborn


National Test - Pediatrics                                                             หนาที่ 19             National Test - Pediatrics                                                หนาที่ 20
12/41
           23. เด็กปากแหวง เพดานโหว นิ้วเกิน คิดถึง syndrome ?                                     02 ID
                      1. Down’s                              2. Patau’s                              27. เด็กอายุ 18 เดือน มีปูเปน TB รักษากิน anti-TB มา 2 สัปดาห เด็กไมมีอาการ ตรวจ
                      3. Turner’s                            4. Edward’s                             รางกายปกติ CXR ปกติ PDD 15 mm. ใหการรักษาที่เหมาะสม
                      5. Klinefelter’s                                                                          1. INH 9 mo                              2. INH + rifampin 6 mo
           24. หญิงอายุ 23 ป G1 GA 42 weeks มีอาการเจ็บครรภ น้ําเดินมา 2 ชั่วโมง พบวามีขี้เทาปน   28. เด็ก 2 ป มาคลินิกเด็กสบาย ได vaccine MMR*1, HBV*3, OPV*3, DPT*3, JE*2 ครั้ง
           ในน้ําคร่ํา มี fetal distress FHR 80 /min แพทยทํา emergency C/S ออกมาพบไดทารก BW        นี้จะใหอะไร
           3,000 g Apgar score 2, 3 ที่ 1 และ 5 นาที ตอมามีเขียว หายใจลําบาก และซีด ถามวาพบ                   1. OPV, DTP                              2. Hib , JE
           ความผิดปกติใดไดในทารกรายนี้                                                                         3. OPV, DPT, JE                          4. OPV, DPT, Hib
                      1. anemia                              2. thrombocytosis                                  5. OPV, DPT, JE, Hib
                      3. hyperactive bowel sound             4. hypotension                          29. เกี่ยวกับ roseola infantum ขอใดผิด
                      5. pulmonary arterial hypertension                                                        1. ผื่นขึ้นวันที่ 3                      2. เกิดจาก HSV 16
           25. เด็กแรกเกิด หนัก ... (รูสึกวาจะหนักปกติ) แมคลอด no complication แรกเกิด มี Hct                3. ผื่นขึ้นที่ลําตัวกอน
           68%, glucose 55 mg/dl จะ management อยางไร                                               30. เด็กเลนน้ํา มี stiff neck มีไข เกิดจากเชื้อใด
                      1. early feeding                       2. 5% D N/2 6 mg/kg/day IV                         1. Naegleria fowleri
                      3. 10% D N/2 2 mg IV push ชาๆ         4. Hct ซ้ําในอีก 6 ชั่วโมง              31. เด็ก 7 ป ~ 3 วันกอน อาเจียนมาก ไมแนใจวามีไขหรือเปลา, 1 วันกอน อาเจียนเปน
                      5. exchange transfusion                                                        เลือด 2 ครั้ง ตรวจรางกาย BP ต่ําๆ , pulse เร็วเล็กนอย , CBC ซีด ,WBC ~2,000 ,
           26. ด็กชาย Term 3,000 g normal labour 26hr หลังคลอด แมเลือด Group O Rh +ve ลูก           Lymphocyte เดน, Plt. ต่ํา, prolong PT /PTT , fibrinogen 120 (200-400) ควรตรวจอะไร
           Group A Rh +ve CBC Hct 40% WBC 11,000,Platelet 260,000 Peripheral blood smear             เพื่อวินิจฉัยภาวะแทรกซอนในรายนี้
           microspherocyte 2+, reticulocyte 10%, Coomb’s test +ve                                               1.D-dimer                                2.Bleeding time
           ทําอยางไร                                                                                           3.Euglobulin lysis time                  4.Clot generation time
                      1. single photo Tx                                                             32. เด็กอายุ 1 ป มีไขสูง 39.5°C ความดันโลหิต 80/60 mmHg, P 140/min มี petechiae ที่
                      2. double photo Tx                                                             แขนและขา
                      3. Intensive photo Tx                                                          จะ Management อยางไร
                      4. partial plasma transfusion                                                             1.Dextran                                2.Dopamine
                      5. total plasma transfusion                                                               3.Platelet                               4.NSS
                                                                                                                5.FFP

National Test - Pediatrics                                                       หนาที่ 21              National Test - Pediatrics                                                   หนาที่ 22
13/41
           33. เด็กหญิง มีไข ปวดหูขวา ได amoxy syr 2 day ไขไมลด ปวดหูมากขึ้น การปฏิบัติที่เห   37. ผูปวยหญิงอายุ 16 ป เมื่อ 4 สัปดาหกอนเปนไข เจ็บคอ ซื้อยากินเอง 2 วัน อาการดีขึ้น
           มะสมตอไปคือ                                                                            ตอมามีปวดเขา 2 ขาง ปวดขอเทาซาย T 38°, no pharyngeal injection, swelling and
                    1. oral cefaclor                     2. oral amoxy/clav                        tenderness out both knee and Lt.ankle, pericardial mb, subcutaneous nodule at
                    3. oral erythromycin                 4. iv ampicillin                          Lt.forearm จงใหการวินิจฉัย
                    5. tympanosynthesis                                                                       1. Acute rheumatic fever               2. Gonococcal arthritis
                                                                                                              3. SLE
           02 Nutrition                                                                            38. เด็กชายอายุ 5 ป แข็งแรงดี มาฉีดวัคซีนตามเกณฑ รางกายแข็งแรง PE no cyanosis,
           34. แมพาบุตรอายุ 7 เดือน มาตรวจ ชันคอไดแตยังไมคว่ํา กระหมอมหนายังไมปด มีเด็ก    normal S1, widely fixed split S2 grade II/VI systolic murmur at left upper sternal border
           ในหมูบานเปนหลายคน มาตรการปองกันคือ                                                  diagnosis?
                     1. เสริม iodine ใหหญิงตั้งครรภ ทุกคนในหมูบาน                                         1. innocent murmur                     2. pulmonary stenosis
                     2. ให thyroid hormone แกเด็กแรกเกิดทุกคน                                               3. aortic stenosis                     4. ASD
                     3. สงเสริมใหกินเกลือผสม iodine ทั้งหมูบาน                                            5. VSD
                     4. เสริมเกลือ iodine ในโรงเรียน                                               39. เด็กอายุ10ป มีประวัติเปนผนังกั้นหองหัวใจพิการแตกําเนิดมีอาการเขียว และเคยหอบ
                     5. ให thyroid hormone แกหญิงตั้งครรภทุกคน                                  เหนื่อยตั้งแตอายุ 2-3ป ตรวจรางกายพบ diasteric blowing murmur ,clubbing finger ถามวา
           02 GI                                                                                   อาการเขียวเกิดจากอะไร
           35. เด็กอายุ 2 ป 6kg ยาว 55 cm AF 3x4 cm PF ยังไมปด ยังชันคอไมได ขยับแขนขาได                 1. Eisenmewnger                        2. Hypoxic spell
           ซีด ทองผูก ถาย 2-3 วัน/ครั้ง อึเปนเม็ดกระสุน                                         40. เด็กทารกอายุ 2 วัน คลอดที่บาน มาดวยเหนื่อย เขียว
                     1. Hypothyroidism                     2. Functional constipation              PE : พบ floppy, cyanosis, HR 50 /min, RR 100 /min ควรทําอะไรเปนอันดับแรก
                     3. Hirschsprung‘s disease             4.Iron def anemia                                  1. Glucose                             4. Adrenaline
           36. เด็ก NL apgar 9,10 แมฝากครรภปกติ Suction ไดเลือด 5 cc สงตรวจอะไร                           2. NSS                                 5. Chest compression
           1. Apt test                                                                                        3. PPV
           2. Coag
           3. CBC                                                                                  02 Critical care
                                                                                                   41. เด็ก 8 ป DKA BP 90/70 poor perfusion Na 145, K 3, Cl 110, HCO3 10, urine ketone
           02 Cardio                                                                               4+ ให fluid ใน 1 hr. แรกเทาไร
                                                                                                              1. 0.9 % NaCl 20 ml/kg             2. 0.45 % NaCl 20 ml/kg


National Test - Pediatrics                                                      หนาที่ 23              National Test - Pediatrics                                                      หนาที่ 24
14/41
                     3. RLS 20 ml/kg                          4. 0.9 % NSS + KCl 20 mEq/kg                       1. ขอความยินยอมพอแมเพื่อ LP
                     5. 0.45 % NSS + KCl 20 mEq/kg                                                               2. LP เลยเพราะเรงดวน
           42. เด็กอายุ 2ป หายใจลําบาก เปนทันทีขณะเลนอยูกับพี่ชาย PE: T 37c RR40/min PR                      3. CT brain
           140/min                                                                                               4. ใหยากันชัก, ATB หลัง H/C
           Lung : fair air entry, Inspiratory+expiratory stridor, suprasternal+subcostal retraction              5. สงตอ
           ถาม Mx                                                                                     46. เด็กหญิงอายุ...ป มีอาการเหมอลอย พัฒนาการปกติ สติปญญาปกติ ตรวจคลื่นไฟฟา
                     1. Back blow-chest thrust                2. Abdominal thrust                     สมองพบ 3 Hz wave ใน hyperventilation การรักษาที่เหมาะสมที่สุดในผูปวยรายนี้คือ
                     3.Heimleich maneuver                     4. ET tube                                         1. ACTH                                 2. Phenytoin
                     5. Bronchoscopy                                                                             3. Vigabatrin                           4. Valproic acid
           43. เด็กชาย 1 ป ไขสูง ซึม มา 1 วัน ตรวจรางกายพบ T 39.2 C, BP 80/60 mmHg, RR 42                     5. Phenobarbital
           /min, P 140 bpm มี petichiae + purpura at both lower extremities อะไรที่ควรให             47. เด็ก ชักมานาน 30 นาที ไมรูสึกตัว ที่ ER ยังชักอยู เปน generalized tonic clonic seizure
                1. Dextran                                                                            ควรใหยาใด
                2. Doputamine                                                                                    1. Diazepam                             2. Phenytoin
                3. Platelet conc.                                                                                3. Valproic acid                        4. Phenobarbital
                4. FFP                                                                                           5. Carbamazepine
                5. NSS
                                                                                                      02 Hemato
           02 Neuro                                                                                   48. ด.ช. 3 ป มีไขมา 1 วัน
           44. เด็กชาย 5 เดือน มีอาการผงกศีรษะและเกร็งแขนขาทั้งสองขางเปนชุดๆ ชุดละ 2-3 นาที         PE: mildly pale, moderate icteric sclera, liver 2 cm below RCM, spleen just palpable
           มักเปนตอนเชาหลังตื่นนอน no eye contact, poor neck control, Spastic tone of all           CBC: Hb 7 Hct 22 wbc 45,000 (N 80 L 40??) platelet ปกติ
           extremities, DTR 1+ all จะใหการรักษาอยางไร                                               PBS: microcytic 2+, hypochromic 2+, anisocytosis 2+, poikilocytosis 2+, schistocytosis
                     1. diazepam                          2. phenytoin                                1+, few ovalocyte and target cell
                     3. Phenobarbital                     4. สงกระตุนพัฒนาการ                       จะสงตรวจ Investigation อะไรเพื่อชวยในการวินิจฉัย
                     5. สงตอกุมารแพทย neuro                                                                  1. D-dimer                               2. G6PD level
           45. เด็กอายุ 8 เดือน แมพามาดวยอาการไขสูง , anterior fontanel โปง , Brudzinski                    3. serum ferritin                        4. Hb typing
           positive จะทําอยางไร

National Test - Pediatrics                                                          หนาที่ 25             National Test - Pediatrics                                                            หนาที่ 26
15/41
        49. เด็กอายุ 9 เดือน มารดาเลี้ยงดวยนมขนตั้งแต 4 เดือน แมพามาฉีดวัคซีน มีซีดเล็กนอย            1. NG lavage                           2. Atropine
        MCV 67,microcytic 1+, anisocytosis 2+, WBC& Plt. ปกติ สาเหตุเกิดจากอะไร                            3. Naloxone                            4. Activated charcoal
                   1. Thalassemia Hb H disease            2. Iron deficency
                   3. Folate deficiency                   4. G-6PD deficiency                     02 Immuno
                   5. Hb E                                                                        55. เด็ก 10 ป เปนโรคหืดมา 5 ป รักษาไมสม่ําเสมอ มีอาการเดือนละหลายครั้ง ครั้งนี้
        50. เด็ก 4 ขวบ ซึม ant fontanelle bulging ชัก กินแตนมขนหวาน                             เหนื่อย หายใจลําบาก กินยาขยายหลอดลมไป 1 เม็ดเมื่อ1 ชม.กอน อาการไมดีขึ้น ที่รพ.
                   1. Arginine prothrombin complex deficiency                                     ไดพนยา Salbutamol ไป 2 doses ยังมีอาการหอบเหนื่อยอยู ควรทําอยางไรตอไป
        51. เด็กชายอายุ 1 ป ไขมา 2 วัน                                                                     1. NB corticosteroid                  2. IV corticosteroid
        PE : moderately pallor, mild jaundice, tachycardia, no hepatosplenomegaly                            3. NB theophylline                    4. NB ipratropium
        CBC: Hct 24% WBC 115,000(N 76% L21%) Plt. 470,000                                         56. เด็ก 3 เดือน disseminater BCG infection อยูรพ. ติด varicella PE: vesicle, pneumonia
        ให bloodsmear มาพบ contracted Hb                                                         น้ําหนักนอย ภาวะภูมิคุมกันผิดปกติจากอะไร
        Diagnosis?                                                                                           1. T-cell                             2. Phagocytosis
                   1. Hb H disease                        2. G6PD deficency                                  3. Opsonization                       4. Ig
                   3. hereditary spherocytosis            4. AIHA                                            5. complement
        52. เด็กชาย 5 ป มีจ้ําเขียวตามแขนขา PE อื่นๆ ปกติ CBC E13% Plt. 180,000 Dx
                   1. Hemophillia                         2. vWD                                  02 Ambulatory
                   3. Hypereosinophil                     4. ITP                                  57. เด็ก 8 เดือน มี labia minora ติดกัน เห็น clitoris ปกติ ไมเห็น hymen/vagina ปสสาวะ
                   5. APDE                                                                        อุจจาระ ปกติ
        53. เด็กหญิง ซีด เหลืองเล็กนอย ตับไมโต มามโต MCV 62 ถาม Dx                             จะทําการรักษาอยางไร
                   1. Thalassemia                         2. G-6-PD deficency                                1. ทา estrogen ที่ labia minora
                   3. Iron deficiency                     4. AIHA                                            2. ทา ATB ที่ labia minora
                   5. HS                                                                                     3. ผาตัดเปด
        02 Toxic                                                                                             4. รักษาความสะอาด แลว ติดตามตอน 2 ป
        54. เด็กอายุ 5 ป กินสารไมทราบชนิด มี lethargy, T 37 c, BP 80/50 mmHg, RR 20 /min,                  5. ไมตองรักษา คอยมาหาถามีปญหาประจําเดือนไมมา
        PR 130 /min, secretion from mouth, pupil 1 mm, มี sweating ควรใหการ management
        อยางไร

National Test - Pediatrics                                                           หนาที่ 27          National Test - Pediatrics                                                          หนาที่ 28
16/41
                                                                                                                1. ตรวจ thalassemia
            03 OB                                                                                    6. ผูหญิง G2 GA 25 wk มี Hx ลูกคนแรกแทงจากการบวมน้ํา จะตรวจวาลูกในทอง มีภาวะ
            1. หญิงไทย 28 yr G1P1 ใหนมลูก 6 wk ตองการคุมกําเนิด 2-3 yr มักมีอาการปวดทองนอย       บวมน้ําไดอยางไร
            เวลามีประจําเดือน ควรใชยาคุมอะไร                                                                   1. amniocentesis                        3. ตรวจเลือดแมจาก fetal cells
                       1. Combinded pill                                                                        2. chordocentesis                       4. Ultrasound
                       2. Pregestin only pill                                                        7. monitor ได variable deceleration ถาม Mx
                       3. DMPA                                                                                  1. induction of labor                   2. augmentation of labor
            2. คลํา sacrumpromontery ไมได ischial spine หางกัน 11 cm. Interbulous หางกัน 8 cm.              3. c/s
            ประมาณวาเวลาผานไป มดลูกหดตัวดี แตปากมดลูกเปดเทาเดิม มีความผิดปกติที่ใด              8. GA 20 weeks มีกลิ่นและคันที่ชองคลอดมา 2 สัปดาห ตรวจรางกายพบวา แดงที่ปาก
                       1. midplane                                                                   ชองคลอด ตกขาวสีเหลืองเขียว มีฟอง ชองคลอดแดง cervix สีแดงคล้ํา (ไมบอกวามี curd)
                       2. outlet                                                                     จะใหยาอะไรรักษา
                       3. inlet                                                                                 1. oral doxycycline                     2. oral tinidazole
            3. หญิงไทยอายุ 30 ป G1P0 GA 8 wk มาดวยเลือดออกกะปริดกะปรอยทางชองคลอดมา                           3. oral ketoconazole                    4. oral fluconazole
            2 วัน มีใจสั่น คลื่นไสอาเจียนมาก ไมมีอาการปวดทอง ตรวจรางกายพบ not pale , uterus                 5. clotrimazole Vg
            at pubic symphysis, BP 140/90, P 110/min, RR 20/min, urine sugar-neg , urine protein-    9. ผูปวยหญิง GA 41 wk ครรภที่ 4 มาดวย labour pain ได drip oxytocin แลว uterine
            neg จงใหการวินิจฉัยที่เปนไปไดมากที่สุด                                                contraction นาน 4 hr 35 min ตอมาเจ็บทองมากขึ้น มดลูกแข็งตัว คอดตรงกลาง FHR 140
                       1. Twins                    4. Trophoblastic disease                          m/ จงใหการวินิจฉัย
                       2. Thyrotoxicosis           5. Pregnancy with myoma uteri                                1. Hyperstimulation uterus              2. Treaten rupture uterus
                       3. preeclampsia                                                               10. หญิงอายุ 34 ป GA 34 wk มาดวยมีเลือดออกทางง vagina เปนเลือดปนกอนเลือด
            4. หญิงไทย G2 มาฝากครรภ GA 18 wk ตรวจเลือดแม A- พอ B+ (อาจสลับกัน ประมาณ              painless, no uterine contraction , เด็กไมดิ้น จงใหการ management
            นี้) ถามทําไรตอ                                                                                    1. CBC                                  2. Coagulogram
                       1. Anti D                             4. Cordocentesis                                   3. Ultrasound                           4. Non stress test
                       2. direct comb                        5. Amniocentesis                                   5. Per vaginal examination with sterile speculum
                       3. indirect coomb                                                             03 Gyne
            5. ผูปวยหญิงมาฝากครรภครั้งแรก ตรวจ osmotic fragility test positive ตองตรวจอะไร       11. การใชยา emergency contraception
            เปนอันดับแรก                                                                                       1.ขนาด 60 2 เม็ด พรอมกัน


National Test - Obstetrics and Gynecology                                           หนาที่ 29           National Test - Obstetrics and Gynecology                                     หนาที่ 30
17/41
                2.ขนาด 100 2 เม็ดหางกัน 12 ชม.                                                   2. Endometrial Bx
                3.ขนาด 60 2 เม็ดหางกัน 12 ชม.                                                    3. Fractional and curettage
                4.ขนาด 100 และ 60 2 เม็ดหางกัน 12 ชม.
      12. หญิง painless ulcer 1 cm ที่ปากชองคลอด elevated border, yellow vaginal discharge
      ถาม Ix
                1. VDRL                       4. G/S
                2. Culture                    5. Biopsy
                3. AFB
      13. หญิงอายุ 34 ป GA 36wk มีอาการปสสาวะแสบขัด และเจ็บที่บริเวณชองคลอด ตรวจ
      รางกายพบ vesicular lesion with shallow ulcer at vulva จงใหการวินิจฉัย
                1. Herpetic vulva                                3. Candidiasis at vulva
                2.Molluscum contagiosum                          4. Syphillis
      14. หญิงอายุ 35 ป น้ําหนัก 70 kg สูง 155 cm มีประจําเดือน 2-3 เดือนตอครั้ง ครั้งนี้
      ประจําเดือนขาดไป 2 เดือน ตอมามีเลือดออกปริมาณมาก นาน 10 วัน ปฏิเสธประวัติ
      เพศสัมพันธ PE ปกติ สาเหตุของประจําเดือนผิดปกติในรายนี้คือ
                1. anovulation                                   4. coagulation defect
                2. irregulartory shedding                        5. corpus luteal insufficiency
                3. irregulation ripening
      15. หญิงอายุ 27 ประจําเดือนคลาดเคลื่อนมา 3 wk มีเลือดออกกระปดกระปรอยมา 4 วัน
      มดลูกโตเล็กนอย PV: พบเลือด os close ไมพบกอนที่ adnexa UPT post. สงอะไร DX
                1. Pap-smear                            4. Transvaginal U/S
                2. AFP                                  5. Transabd U/S
                3. Serum Beta-HCG
      16. ญ หมดประจําเดือนแลว มีเลือดออกทางชองคลอดประมาณ 2 wks U/S พบ
      endometrium หนา 2 mm ควรทําอยางไร
                1. Hysterectomy


National Test - Obstetrics and Gynecology                                          หนาที่ 31      National Test - Obstetrics and Gynecology   หนาที่ 32
18/41
        04 GenSx                                                                               6. ผูปวยหญิงไทย อายุ 56 ป ปวดทองดานบนขวา 2 วัน มีไข ไมคลื่นไสอาเจียน มีประวัติ
        1. ผูปวยชายมีไข หนาวสั่น ปวดทอง RLQ กดเจ็บ U/S พบ CBD dilate, multiple             ปวดเปนๆหายๆ มา 2 ป ตรวจรางกาย : T 39, HR…, RR…, Abdomen: soft tenderness at
        hypoechoic mass at segment 6                                                           RUQ investigations : TB 5.8, DB 4.2, SGOT 9, SGPT 120, DLP 168 จะใหยาอะไร
                   1.Hepatoma                               3.Amebic liver abscess                        1. Ciprofloxacin
                   2.CholangioCA                            4.pyogenic liver abscess                      2. Ceftriaxone + …
                   5. Infected Cyst                                                                       3. Ceftriaxone + Metronidazole
        2. เด็กชาย อายุ 5 ปปวดทองมาก ทองเสีย 2 ครั้ง อาเจียน 3 ครั้ง ปวดรอบ umbulicus+RUQ              4. Ampicilin + Gentamycin
        สง investigate อะไร                                                                              5. Gentamycin + Metronidazole
                   1.lipase                                 4.CBC                              7. ผูปวยชายอายุ 40 ป มีประวัติกอนเขาๆ ออกๆ ที่อัณฑะ ตอนปวด บวม แดง กอนออกมา
                   2.amylase                                5.U/A                              แลว จงใหการวินิจฉัย
                   3.film acute abdomen series                                                            1. orchitis                   2. torsion testis
        3. ชาย 55 ป ดื่มสุรา cirrhosis มา7 ป, U/S -liver:solid, ill-defined mass 7 cm                   3. hydrocelitis               4. incarcerated hernia
        Investigate?                                                                           8. ผูปวยหญิงอายุ 52 ป เปน cholecystitis ไมไดรับการรักษา 4 วันกอนมีอาการปวดทอง
                   1.CEA                         4.AFP                                         บริเวณลิ้นป จะใหนอนแนนมากขึ้น มีปวดราวไปหลัง อาเจียนเปนอาหาร ตรวจรางกายพบ
                   2.CA19-9                      5.ALP                                         v/s stable มี guarding and rebound tenderness at upper abdomen จงใหการวินิจฉัย
                   3.Beta-hCG                                                                             1. acute cholecystitis        2. acute appendicitis
        4. หญิงไทย 25 ป กินน้ํายาลางหองน้ํามา ผลแทรกซอนระยะยาว                                        3. peptic perforate           4. acute pancreatitis
                   1. esophagitis                4. chemical pneumonitis                                  5. small bowel obstruction
                   2 mucositis                   5. stricture esophagus
                   3. mediastinitis                                                            04 HNB
                                                                                               9. เด็กหญิงอายุ 9 ป มารดาพามาดวยเรื่องกอนที่เตานม กดเจ็บ P.E. พบ mass เสนผาน
                                                                                               ศูนยกลาง 1.5 cm under areolar, tender lump, no lymphadenopathy อื่นๆปกติ จงDx
        5. ผูปวยหญิงทองผูก+ปวดทอง RLQ film เจอ …………………ถามวา management คือ                         1. breast bud                 4. gynecomastia
                   1. CT abdomen                                                                        2. fibroadenoma               5. breast abscess
                   2. Diverticulectomy                                                                  3. fat lump
                   3. F/U Barium enema ทุกป


National Test - Surgery                                                     หนาที่ 33                National Test - Surgery                                                       หนาที่ 34
Nt2009 complete all
Nt2009 complete all
Nt2009 complete all
Nt2009 complete all
Nt2009 complete all
Nt2009 complete all
Nt2009 complete all
Nt2009 complete all
Nt2009 complete all
Nt2009 complete all
Nt2009 complete all
Nt2009 complete all
Nt2009 complete all
Nt2009 complete all
Nt2009 complete all
Nt2009 complete all
Nt2009 complete all
Nt2009 complete all
Nt2009 complete all
Nt2009 complete all
Nt2009 complete all
Nt2009 complete all
Nt2009 complete all

More Related Content

What's hot

Osce examination si116
Osce examination si116Osce examination si116
Osce examination si116vora kun
 
Nt2553step3round1 28NOV2553
Nt2553step3round1 28NOV2553Nt2553step3round1 28NOV2553
Nt2553step3round1 28NOV2553vora kun
 
NTstep3round2 9_jan2554
NTstep3round2 9_jan2554NTstep3round2 9_jan2554
NTstep3round2 9_jan2554vora kun
 
สอบ-ศรว-มีนาคม-2551
สอบ-ศรว-มีนาคม-2551สอบ-ศรว-มีนาคม-2551
สอบ-ศรว-มีนาคม-2551rookiess
 
Step3 Tutorial by SWU book1
Step3 Tutorial by SWU book1Step3 Tutorial by SWU book1
Step3 Tutorial by SWU book1vora kun
 
Osce ศรว ครั้งที่สอง 10jan53
Osce ศรว ครั้งที่สอง 10jan53Osce ศรว ครั้งที่สอง 10jan53
Osce ศรว ครั้งที่สอง 10jan53vora kun
 
Case study surgery
Case study surgeryCase study surgery
Case study surgerysoftmail
 
Osce คณะ si 115
Osce คณะ si 115Osce คณะ si 115
Osce คณะ si 115vora kun
 
การซักประวัติการเจ็บป่วย
การซักประวัติการเจ็บป่วยการซักประวัติการเจ็บป่วย
การซักประวัติการเจ็บป่วยOzone Thanasak
 
2007821172158 466 6438_1
2007821172158 466 6438_12007821172158 466 6438_1
2007821172158 466 6438_1New Srsn
 

What's hot (16)

Osce examination si116
Osce examination si116Osce examination si116
Osce examination si116
 
Nt2553step3round1 28NOV2553
Nt2553step3round1 28NOV2553Nt2553step3round1 28NOV2553
Nt2553step3round1 28NOV2553
 
NTstep3round2 9_jan2554
NTstep3round2 9_jan2554NTstep3round2 9_jan2554
NTstep3round2 9_jan2554
 
สอบ-ศรว-มีนาคม-2551
สอบ-ศรว-มีนาคม-2551สอบ-ศรว-มีนาคม-2551
สอบ-ศรว-มีนาคม-2551
 
Step3 Tutorial by SWU book1
Step3 Tutorial by SWU book1Step3 Tutorial by SWU book1
Step3 Tutorial by SWU book1
 
Example osce
Example osceExample osce
Example osce
 
Osce ศรว ครั้งที่สอง 10jan53
Osce ศรว ครั้งที่สอง 10jan53Osce ศรว ครั้งที่สอง 10jan53
Osce ศรว ครั้งที่สอง 10jan53
 
Key word osce
Key word osceKey word osce
Key word osce
 
National license 2010 by med tu 16
National license 2010 by med tu 16National license 2010 by med tu 16
National license 2010 by med tu 16
 
National test _2553_TU
National test _2553_TUNational test _2553_TU
National test _2553_TU
 
Case study surgery
Case study surgeryCase study surgery
Case study surgery
 
Case study : dengue fever
Case study : dengue feverCase study : dengue fever
Case study : dengue fever
 
Osce คณะ si 115
Osce คณะ si 115Osce คณะ si 115
Osce คณะ si 115
 
การซักประวัติการเจ็บป่วย
การซักประวัติการเจ็บป่วยการซักประวัติการเจ็บป่วย
การซักประวัติการเจ็บป่วย
 
2007821172158 466 6438_1
2007821172158 466 6438_12007821172158 466 6438_1
2007821172158 466 6438_1
 
Nl part ii march 2009
Nl part ii march 2009Nl part ii march 2009
Nl part ii march 2009
 

Viewers also liked

Survivor NT step2 SIRIRAJ book 1
Survivor NT step2 SIRIRAJ book 1Survivor NT step2 SIRIRAJ book 1
Survivor NT step2 SIRIRAJ book 1vora kun
 
ศรว 51 By Cmu
ศรว 51 By Cmuศรว 51 By Cmu
ศรว 51 By Cmuvora kun
 
Step3 Tutorial by SWU book2
Step3 Tutorial by SWU book2Step3 Tutorial by SWU book2
Step3 Tutorial by SWU book2vora kun
 
Clinical medicine
Clinical medicineClinical medicine
Clinical medicineMoni Buvy
 
Survival for all draft 1 - 3
Survival for all draft 1  - 3Survival for all draft 1  - 3
Survival for all draft 1 - 3Domo Kwan
 
Survivor NT step2 SIRIRAJ book 2
Survivor NT step2 SIRIRAJ book 2Survivor NT step2 SIRIRAJ book 2
Survivor NT step2 SIRIRAJ book 2vora kun
 
Abnormal pap smear ศิริราช ppt
Abnormal pap smear ศิริราช pptAbnormal pap smear ศิริราช ppt
Abnormal pap smear ศิริราช pptvora kun
 
ศรว 51 ANS By Cmu
ศรว 51 ANS By Cmuศรว 51 ANS By Cmu
ศรว 51 ANS By Cmuvora kun
 
Nle step 2_2009 si115-116 and nle_step_2_2009 nctms editors cut key
Nle step 2_2009 si115-116 and nle_step_2_2009 nctms editors cut keyNle step 2_2009 si115-116 and nle_step_2_2009 nctms editors cut key
Nle step 2_2009 si115-116 and nle_step_2_2009 nctms editors cut keyLoveis1able Khumpuangdee
 
NT step2 march 53
NT step2 march 53NT step2 march 53
NT step2 march 53vora kun
 
ortho 02 orthopaedic complication & prevention + orthopaedic trauma (practica...
ortho 02 orthopaedic complication & prevention + orthopaedic trauma (practica...ortho 02 orthopaedic complication & prevention + orthopaedic trauma (practica...
ortho 02 orthopaedic complication & prevention + orthopaedic trauma (practica...vora kun
 
CPR 2010 อ ปริญญา รามา
CPR 2010 อ ปริญญา รามาCPR 2010 อ ปริญญา รามา
CPR 2010 อ ปริญญา รามาvora kun
 
ประชุมวิชาการ ศิริราช 52
ประชุมวิชาการ ศิริราช 52ประชุมวิชาการ ศิริราช 52
ประชุมวิชาการ ศิริราช 52vora kun
 
ประชุมวิชาการ ศิริราช 53
ประชุมวิชาการ ศิริราช 53ประชุมวิชาการ ศิริราช 53
ประชุมวิชาการ ศิริราช 53vora kun
 
Mdcu Preventive Screening
Mdcu Preventive ScreeningMdcu Preventive Screening
Mdcu Preventive Screeningvora kun
 
Endocrine Med 2010 Step2
Endocrine Med 2010 Step2Endocrine Med 2010 Step2
Endocrine Med 2010 Step2vora kun
 

Viewers also liked (20)

Survivor NT step2 SIRIRAJ book 1
Survivor NT step2 SIRIRAJ book 1Survivor NT step2 SIRIRAJ book 1
Survivor NT step2 SIRIRAJ book 1
 
รวมข้อสอบCompre nl
รวมข้อสอบCompre nlรวมข้อสอบCompre nl
รวมข้อสอบCompre nl
 
ศรว 51 By Cmu
ศรว 51 By Cmuศรว 51 By Cmu
ศรว 51 By Cmu
 
Step3 Tutorial by SWU book2
Step3 Tutorial by SWU book2Step3 Tutorial by SWU book2
Step3 Tutorial by SWU book2
 
Clinical medicine
Clinical medicineClinical medicine
Clinical medicine
 
Survival for all draft 1 - 3
Survival for all draft 1  - 3Survival for all draft 1  - 3
Survival for all draft 1 - 3
 
Survivor NT step2 SIRIRAJ book 2
Survivor NT step2 SIRIRAJ book 2Survivor NT step2 SIRIRAJ book 2
Survivor NT step2 SIRIRAJ book 2
 
Abnormal pap smear ศิริราช ppt
Abnormal pap smear ศิริราช pptAbnormal pap smear ศิริราช ppt
Abnormal pap smear ศิริราช ppt
 
ศรว 51 ANS By Cmu
ศรว 51 ANS By Cmuศรว 51 ANS By Cmu
ศรว 51 ANS By Cmu
 
Drug for-int
Drug for-intDrug for-int
Drug for-int
 
Nle step 2_2009 si115-116 and nle_step_2_2009 nctms editors cut key
Nle step 2_2009 si115-116 and nle_step_2_2009 nctms editors cut keyNle step 2_2009 si115-116 and nle_step_2_2009 nctms editors cut key
Nle step 2_2009 si115-116 and nle_step_2_2009 nctms editors cut key
 
National licensce ii 2554 print meng
National licensce ii 2554 print mengNational licensce ii 2554 print meng
National licensce ii 2554 print meng
 
NT step2 march 53
NT step2 march 53NT step2 march 53
NT step2 march 53
 
Swu
SwuSwu
Swu
 
ortho 02 orthopaedic complication & prevention + orthopaedic trauma (practica...
ortho 02 orthopaedic complication & prevention + orthopaedic trauma (practica...ortho 02 orthopaedic complication & prevention + orthopaedic trauma (practica...
ortho 02 orthopaedic complication & prevention + orthopaedic trauma (practica...
 
CPR 2010 อ ปริญญา รามา
CPR 2010 อ ปริญญา รามาCPR 2010 อ ปริญญา รามา
CPR 2010 อ ปริญญา รามา
 
ประชุมวิชาการ ศิริราช 52
ประชุมวิชาการ ศิริราช 52ประชุมวิชาการ ศิริราช 52
ประชุมวิชาการ ศิริราช 52
 
ประชุมวิชาการ ศิริราช 53
ประชุมวิชาการ ศิริราช 53ประชุมวิชาการ ศิริราช 53
ประชุมวิชาการ ศิริราช 53
 
Mdcu Preventive Screening
Mdcu Preventive ScreeningMdcu Preventive Screening
Mdcu Preventive Screening
 
Endocrine Med 2010 Step2
Endocrine Med 2010 Step2Endocrine Med 2010 Step2
Endocrine Med 2010 Step2
 

Similar to Nt2009 complete all

Hemorrhagic shock 15 พค.2558
Hemorrhagic shock 15 พค.2558Hemorrhagic shock 15 พค.2558
Hemorrhagic shock 15 พค.2558Krongdai Unhasuta
 
Exercise national license_part_ii_march_2009_2
Exercise national license_part_ii_march_2009_2Exercise national license_part_ii_march_2009_2
Exercise national license_part_ii_march_2009_2Loveis1able Khumpuangdee
 
Motor weakness and Cerebrovascular Disease
Motor weakness and Cerebrovascular DiseaseMotor weakness and Cerebrovascular Disease
Motor weakness and Cerebrovascular DiseaseNarongrit Kasemsap
 
กาย่า presentation
กาย่า presentationกาย่า presentation
กาย่า presentationCrystalpet
 
Interesting case ACLS.pdf
Interesting case ACLS.pdfInteresting case ACLS.pdf
Interesting case ACLS.pdfStafarne
 
Surgery of acquired heart disease อ.วรวงศ์ ศลิษฏ์อรรถกร
Surgery of acquired heart disease อ.วรวงศ์ ศลิษฏ์อรรถกรSurgery of acquired heart disease อ.วรวงศ์ ศลิษฏ์อรรถกร
Surgery of acquired heart disease อ.วรวงศ์ ศลิษฏ์อรรถกรpohgreen
 
CPG Thai Stroke infarct retrieved since 2555
CPG Thai Stroke infarct retrieved since 2555CPG Thai Stroke infarct retrieved since 2555
CPG Thai Stroke infarct retrieved since 2555Thorsang Chayovan
 
E831 b handout_pharmacotherapeutics_arh_50
E831 b handout_pharmacotherapeutics_arh_50E831 b handout_pharmacotherapeutics_arh_50
E831 b handout_pharmacotherapeutics_arh_50Loveis1able Khumpuangdee
 
E831 b handout_pharmacotherapeutics_arh_50
E831 b handout_pharmacotherapeutics_arh_50E831 b handout_pharmacotherapeutics_arh_50
E831 b handout_pharmacotherapeutics_arh_50Loveis1able Khumpuangdee
 
Cvst 2
Cvst 2Cvst 2
Cvst 2med
 
การดูแลผู้ป่วยแน่นหน้าอก สงสัย Ischemiaxxx
การดูแลผู้ป่วยแน่นหน้าอก สงสัย  Ischemiaxxxการดูแลผู้ป่วยแน่นหน้าอก สงสัย  Ischemiaxxx
การดูแลผู้ป่วยแน่นหน้าอก สงสัย Ischemiaxxxeremslad
 

Similar to Nt2009 complete all (15)

National license 2010 by med tu 16
National license 2010 by med tu 16National license 2010 by med tu 16
National license 2010 by med tu 16
 
Hemorrhagic shock 15 พค.2558
Hemorrhagic shock 15 พค.2558Hemorrhagic shock 15 พค.2558
Hemorrhagic shock 15 พค.2558
 
Case01
Case01Case01
Case01
 
Exercise national license_part_ii_march_2009_2
Exercise national license_part_ii_march_2009_2Exercise national license_part_ii_march_2009_2
Exercise national license_part_ii_march_2009_2
 
Nl part ii march 2009
Nl part ii march 2009Nl part ii march 2009
Nl part ii march 2009
 
Ihd
IhdIhd
Ihd
 
Motor weakness and Cerebrovascular Disease
Motor weakness and Cerebrovascular DiseaseMotor weakness and Cerebrovascular Disease
Motor weakness and Cerebrovascular Disease
 
กาย่า presentation
กาย่า presentationกาย่า presentation
กาย่า presentation
 
Interesting case ACLS.pdf
Interesting case ACLS.pdfInteresting case ACLS.pdf
Interesting case ACLS.pdf
 
Surgery of acquired heart disease อ.วรวงศ์ ศลิษฏ์อรรถกร
Surgery of acquired heart disease อ.วรวงศ์ ศลิษฏ์อรรถกรSurgery of acquired heart disease อ.วรวงศ์ ศลิษฏ์อรรถกร
Surgery of acquired heart disease อ.วรวงศ์ ศลิษฏ์อรรถกร
 
CPG Thai Stroke infarct retrieved since 2555
CPG Thai Stroke infarct retrieved since 2555CPG Thai Stroke infarct retrieved since 2555
CPG Thai Stroke infarct retrieved since 2555
 
E831 b handout_pharmacotherapeutics_arh_50
E831 b handout_pharmacotherapeutics_arh_50E831 b handout_pharmacotherapeutics_arh_50
E831 b handout_pharmacotherapeutics_arh_50
 
E831 b handout_pharmacotherapeutics_arh_50
E831 b handout_pharmacotherapeutics_arh_50E831 b handout_pharmacotherapeutics_arh_50
E831 b handout_pharmacotherapeutics_arh_50
 
Cvst 2
Cvst 2Cvst 2
Cvst 2
 
การดูแลผู้ป่วยแน่นหน้าอก สงสัย Ischemiaxxx
การดูแลผู้ป่วยแน่นหน้าอก สงสัย  Ischemiaxxxการดูแลผู้ป่วยแน่นหน้าอก สงสัย  Ischemiaxxx
การดูแลผู้ป่วยแน่นหน้าอก สงสัย Ischemiaxxx
 

More from vora kun

Thai Osteoporosis guideline 2553
Thai Osteoporosis guideline 2553Thai Osteoporosis guideline 2553
Thai Osteoporosis guideline 2553vora kun
 
ortho 02 orthopaedic complication & prevention + orthopaedic trauma (practica...
ortho 02 orthopaedic complication & prevention + orthopaedic trauma (practica...ortho 02 orthopaedic complication & prevention + orthopaedic trauma (practica...
ortho 02 orthopaedic complication & prevention + orthopaedic trauma (practica...vora kun
 
ortho 06 common ortho dis 2 edited 12 mar 10
ortho 06 common ortho dis 2 edited 12 mar 10ortho 06 common ortho dis 2 edited 12 mar 10
ortho 06 common ortho dis 2 edited 12 mar 10vora kun
 
ortho 05 common rheumatic dx rx
ortho 05 common rheumatic dx rxortho 05 common rheumatic dx rx
ortho 05 common rheumatic dx rxvora kun
 
ortho 01 management of open fracture-update by kk 31052010
ortho 01 management of open fracture-update by kk 31052010ortho 01 management of open fracture-update by kk 31052010
ortho 01 management of open fracture-update by kk 31052010vora kun
 
ortho 04 drugs in orthopaedic (principle & common use)
ortho 04 drugs in orthopaedic (principle & common use)ortho 04 drugs in orthopaedic (principle & common use)
ortho 04 drugs in orthopaedic (principle & common use)vora kun
 
ortho 03 principle of closed reduction in fracture and dislocation
ortho 03 principle of closed reduction in fracture and dislocationortho 03 principle of closed reduction in fracture and dislocation
ortho 03 principle of closed reduction in fracture and dislocationvora kun
 
หัตถการที่จำเป็นทางสูติ
หัตถการที่จำเป็นทางสูติหัตถการที่จำเป็นทางสูติ
หัตถการที่จำเป็นทางสูติvora kun
 
SWU CXR interpretation
SWU  CXR interpretationSWU  CXR interpretation
SWU CXR interpretationvora kun
 
Total parenteral nutrition
Total parenteral nutritionTotal parenteral nutrition
Total parenteral nutritionvora kun
 
NeuroSx step2 Review
NeuroSx step2 ReviewNeuroSx step2 Review
NeuroSx step2 Reviewvora kun
 

More from vora kun (11)

Thai Osteoporosis guideline 2553
Thai Osteoporosis guideline 2553Thai Osteoporosis guideline 2553
Thai Osteoporosis guideline 2553
 
ortho 02 orthopaedic complication & prevention + orthopaedic trauma (practica...
ortho 02 orthopaedic complication & prevention + orthopaedic trauma (practica...ortho 02 orthopaedic complication & prevention + orthopaedic trauma (practica...
ortho 02 orthopaedic complication & prevention + orthopaedic trauma (practica...
 
ortho 06 common ortho dis 2 edited 12 mar 10
ortho 06 common ortho dis 2 edited 12 mar 10ortho 06 common ortho dis 2 edited 12 mar 10
ortho 06 common ortho dis 2 edited 12 mar 10
 
ortho 05 common rheumatic dx rx
ortho 05 common rheumatic dx rxortho 05 common rheumatic dx rx
ortho 05 common rheumatic dx rx
 
ortho 01 management of open fracture-update by kk 31052010
ortho 01 management of open fracture-update by kk 31052010ortho 01 management of open fracture-update by kk 31052010
ortho 01 management of open fracture-update by kk 31052010
 
ortho 04 drugs in orthopaedic (principle & common use)
ortho 04 drugs in orthopaedic (principle & common use)ortho 04 drugs in orthopaedic (principle & common use)
ortho 04 drugs in orthopaedic (principle & common use)
 
ortho 03 principle of closed reduction in fracture and dislocation
ortho 03 principle of closed reduction in fracture and dislocationortho 03 principle of closed reduction in fracture and dislocation
ortho 03 principle of closed reduction in fracture and dislocation
 
หัตถการที่จำเป็นทางสูติ
หัตถการที่จำเป็นทางสูติหัตถการที่จำเป็นทางสูติ
หัตถการที่จำเป็นทางสูติ
 
SWU CXR interpretation
SWU  CXR interpretationSWU  CXR interpretation
SWU CXR interpretation
 
Total parenteral nutrition
Total parenteral nutritionTotal parenteral nutrition
Total parenteral nutrition
 
NeuroSx step2 Review
NeuroSx step2 ReviewNeuroSx step2 Review
NeuroSx step2 Review
 

Nt2009 complete all

  • 1. 1/41 Contents 01 Medicine P. 1 02 Pediatrics P. 16 03 Obstetrics and Gynecology P. 29 04 Surgery P. 33 05 Orthopaedics and rehab P.39 06 Eye and ENT P. 41 07 Psychiatry P. 47 08 Forensic Med and Ethics P. 51 09 Miscellaneous P. 53 ขอบคุณเพื่อนๆ SI 116 ทุกคนที่ชวยกันพิมพขอสอบ ขอบคุณพี่ SI 115 ทุกคนที่ชวยกันจําขอสอบ
  • 2. 2/41 01 Critical care 1. HS 2. G6PD-def 1. ชาย 50 ป หมดสติมา, no pulse, no respiration , ฟง heart sound ไมได แพทยใส tube 3. AIHA 4. Thalassemia with hemolytic crisis ,chest compression ติด EKG ไดดังรูป (เปนรูป V fib) ถาม Mx 5. PNH 1. IV Adenosine 2. IV Epinephrine 5. หญิงอายุ 18 ปเหนื่อยงายมา ? วัน มีซีดปานกลาง เหลืองเล็กนอย ตับไมโต มามโต 2 cm 3. chest compression ตอ 4. defibrillation CBC Hct 20 MCV 62 WBC 95,000 N85 L 15 Plt 250,000 reti 8 ถาม Dx 2. หญิง 65 ป ใจสั่น หมดสติไป 5 นาที ญาตินําสงรพ. รูสติดี .. HR 40 /min EKG ดังรูป 1. Thalassemia 2. Iron Def จงใหการวินิจฉัย 3. G6PD 4. Hereditary spherocytosis 5. AIHA 6. ชายอายุ 50 ป เปน cirrhosis มาดวยอาเจียนเปนเลือด PE : shifting dullness positive, no hepatomegaly, splenomegaly 4 cm CBC : WBC 10,000 , Plt 60,000 , %N 32 , %L 76 จะใหสวนประกอบของเลือดอะไร 1. PRC 2. Cryoprecipitate 1. 3rd heart block 3. Ciyo-less 4. Platelet conc. 2. 2nd degree AV block mobitz I 5. FFP 3. 2nd degree AV block mobitz II 4. 1st degree block 01 Neurology 3. ชายไทยอายุ 23 ป ถูกผึ้งตอยมาดวย wheel 3+, อาเจียน BP 80/50 mmHg PE:edema 7. ผูปวยหญิงมาดวย proximal muscle weakness เปนมากตอนบาย ๆ มี diplopia, หนังตา รอบปาก Management? ตก, DTR 2+ สงตรวจอะไรเพื่อการวินิจฉัย 1. H2 antagonist 2. H1 antagonist 1. LP 2. Prostigmine test 3. Dexamethasone IV 4. Adrenaline IM 3. Electrolyte 4. Muscle biopsy 8. เด็กอายุ 8 เดือน แมพามาดวยอาการไขสูง, anterior fontanel โปง, Brudzinski positive 01 Hemato จะทําอยางไร 4. ผูปวยหญิงอายุ 30 ป เหนื่อยเพลียมา 2 สัปดาห ปสสาวะเขมขึ้น ไมมีไข no 1. ขอความยินยอมพอแมเพื่อ LP hepatosplenomegaly, moderate pale, mild icteric sclera, WBC 3,600, Plt 234,000 , blood 2. LP เลยเพราะเรงดวน smear ดังรูป จงใหการวินิจฉัย 3. CT brain National Test - Medicine หนาที่ 1 National Test - Medicine หนาที่ 2
  • 3. 3/41 4. ใหยากันชัก, ATB หลัง H/C 13. หญิง อายุ 65 ป เปนเบาหวานมา 5 ป เมื่อ 2 วันกอนปสสาวะแสบขัด วันนี้มีไขสูง 5. สงตอ หนาวสั่น ซึมลง, T 39 C BP 140/80 P 120, U/A protein trace, sugar neg, SpGr 1.018, 8. หญิง 27 ป มีอาการปวดศีรษะตุบๆ ปวดขางเดียว เปนสลับขางกันเปนๆหายๆ เดือนละ bacteria numerous, RBC 10-15, WBC 50-60, no cast/ept cell จงใหการรักษา 4-5 ครั้ง ขณะนี้ on oral contraceptive มานาน … ป ตรวจรางกายไมพบ neurological 1. Ampicillin 2. Norfloxacin deficit ,……. 3. Gentamycin 4. Ceftriaxone 1. CT brain 2. MRI brain 5. Cotrimoxazole 3.ให diazepam 4.ให tramadol 14. ชาย 33 ป ปวดศีรษะเรื้อรังมาหลายเดือน ใจสั่น เหนื่อยงาย หนาแดง sweating ปวดหัว 5.ให amitriptyline มากขึ้น BP 180/110, P 110/min flushing การตรวจเพิ่มเติมเพื่อการวินิจฉัยโรค 10. ผูปวยชาย เดินชา มี resting tremor Rt > Lt มี Cogwheel rigidity ใหการรักษาอยางไร 1. Urine VMA 1. Selegiline 2. Levodopa 2. CT brain 3. Benhexole 4. Bromocriptine 3. Serum electrolyte 11. ชายอายุ 24 ป มีอาการชักเกร็งกระตุกทั่วตัว ตาเหลือก ไมรูสึกตัวมานาน 1 ชั่วโมง มี 4. TFT ประวัติโรคลมชักตั้งแตอายุ15ป ชักปละ 3-4 ครั้ง ขาดการรักษามา 1ป ขณะตรวจไมมี 5. IVP อาการชักแลว การรักษาที่เหมาะสมที่สุดในผูปวยรายนี้ 1. Phenytoin 2. Diazepam 15. ชาย 25 ป ปวดศีรษะมานาน 1 mo วันนี้ออนเพลีย แขนขาไมมีแรง BP 190/120 motor 3. Midazolam 4. Phenobarbital power 3/5, P 110 investigate 5. Gabapentin 1. Urine VMA <<<<เฉลย 2. CT brain 01 Nephro 3. Serum electrolyte 12. ผูปวยหญิงไทยอายุ 43 ป แขนขาออนแรงมา 1-3 วัน 1 เดือนกอนมีอาการขาบวม 4. TFT ไปพบแพทยที่คลินิกได hydrochlorothiazide (50 mg) วันละเม็ด อาการดังกลาวเกิดจาก 5. IVP อะไร 1. Hypokalemia 2. Hyponatremia 16. ชายอายุ 39 ป มารพ.ดวย ปสสาวะสีน้ําลางเนื้อมา 2 วัน ตรวจรางกายพบ puffy eyelids, 3. Hyperglycemia 4. Hypocalcemia BP 140/90, UA red brown color urine, Sp.Gr. 1.025 Albumin 1+ RBC 100/HF, WBC 5. Hypomagnesemia 10/HF with rbc cast รักษาอยางไร 1. Norfloxacin 2. Ceftriaxone National Test - Medicine หนาที่ 3 National Test - Medicine หนาที่ 4
  • 4. 4/41 3. Furosemide 4. Prednisolone 5. Congenital heart disease 17. ผูปวยหญิงอายุ _ ป สูบบุหรี่ 30 pack year อาการเขาไดกับ CA Lung, no underlying 21. หญิงอายุ 45 ป มีไขตอนเย็นๆ ไอมีเสมหะ ไมมีเลือดมา 3 wk ได amoxiclav 4 days disease levofloxacin 5 days อาการไมดีขึ้น CXR alveolar infiltration with RUL cavity ถามวา Mx ตรวจ Lab : E’lyte : Na 125, K _ , Cl 97 ; Urine Na 10, Urine osmol 50 ตอไปคือ อะไรคือสาเหตุของhyponatremia ในผูปวยรายนี้ 1. Sputum AFB 2. Bronchoscope 1. SIADH 2. Low salt intake 3. CT chest 3. Renal wasting 4. Thiazide 4. FNA 5. Primary polydipsia 22. คนงานโรงงานกระเบื้อง... ตอมาสงสัยวาเปนโรคมะเร็งปอด ในการวินิจฉัยวา เกิดโรค 18. ผูปวย heart failure มี fine crepitation ปสสาวะไมออก ให furosemide iv 2 doses ยัง จากการประกอบอาชีพ ตองทําอะไรเปนอันดับแรก ปสสาวะไมออกอีก BUN 75,Cr 5.2,CO 12, Na 131,K 6 ควรทําอะไรตอ 1. ตรวจสอบขอมูลเกี่ยวกับการทํางานอยางละเอียด 1. Dialysis 2. IV furosemide 2. วินิจฉัยใหไดวาเปนมะเร็งปอดจริง 3. Ca gluconate 4. Restrict salt diet 3. หาขอมูลวามีหลักฐานสัมพันธกับมะเร็งปอด 5. NaHCO3 4. ตรวจวัดปริมาณสารวาพอจะทําใหเกิดมะเร็งปอดหรือไม 19. ผูปวยหญิงอายุ 43 ป มีอาการออนเพลีย แขนและขาออนแรงมา 2 วัน เมื่อประมาณ 1 5. – เดือนกอนมีอาการขาบวมเล็กนอย ไดรับยา HTCZ (50mg) 1 เม็ดตอวัน รับประทานทุกวัน 23. ผูปวยชายอายุ 50 ป สูบบุหรี่ 2-3 มวน/วัน มานาน 30 ป มีอาการแนนหนาอก เหนื่อย อาการผูปวยเกิดจากเหตุผลใด มา 2 วัน หนาและแขนบวม ตรวจพบ not pale, face & upper extremity edema, superficial 1. Hypo K 2. Hypo Na vein dilatation at chest wall ทํา CXR ผลปกติ สาเหตุที่นาจะเปนไปไดในผูปวยรายนี้ 3. Hyperglycemia 4. Hypo Ca 1. Cor pulmonale 2. Cardiac temponade 5. Hypo Mg 3. Pulmonary tuberculosis 4. Bronchogenic carcinoma 24. ชาย 25 ป 2 สัปดาหกอนดื่มสุรา เมาและอาเจียนมาก และหมดสติ ตรวจพบ pneumonia 01 Res ที่ superior segment of right upper lobe ตรวจเสมหะพบ pleomorphic gram positive cocci 20. ผูหญิงอายุ 20 ป มีไอเรื้อรัง ตั้งแต 10 ปกอน บางครั้งมีเสมหะเขียว ตอนเด็กๆเคยมี vary in size and shape ใหการรักษาดวย co-amoxyclav นาน 3 วัน ยังคงมีไข ตรวจ x-ray Respiratory tract infection เยินๆตอนอายุ 7 ป CXR : peribroncheal thickening เพิ่มเติมพบมี pleural effusion เจาะน้ํามาตรวจได น้ําสีเหลืองขุน WBC 2,500 (N 95%) , จงใหการวินิจฉัย glucose 30 mg/dl , LDH 900 ตรวจ gram stain พบเชื้อแบบเดิม จงบอกการปฏิบัติที่ 1. Lung abscess 2. Bronchiectasis เหมาะสมในผูปวยรายนี้ 3. Pulmonary tuberculosis 4. Chronic bronchitis 1. เพิ่มยา aminoglycosides National Test - Medicine หนาที่ 5 National Test - Medicine หนาที่ 6
  • 5. 5/41 2. เปลี่ยนเปนยากลุม carbapenem 01 Endocrine 3. ใหยาเดิม + CT chest 29. ผูปวยหญิงอายุ 28 ป มีไข ออนเพลีย มา 1 สัปดาห เหนื่อยงาย ใจสั่น PE : T 38 °C, P 4. ใหยาเดิม + ใส ICD 120/min, tremor both hand Neck : marked tender thyroid gland and diffuse 5. ใหยาเดิม + เจาะน้ําในอีก 24 hrs. enlargement จงใหการวินิจฉัย 1. Acute thyroiditis 2. Grave’s disease 01 Toxico 3. Hashimoto’s thyroiditis 4. Suppurative thyroiditis 25. ผูปวยชาย กินยาฆาตัวตายมา มีอาการ sweating CVS: normal , RS: diffuse lung 5. Papillary CA thyroid infiltration , muscle มี fasciculation 30. ผูปวยหญิง อายุ 28 ป มีอาการเหนื่อยมา 1สัปดาห P120/min RR20/min BP120/80 1. Salicylate 2. Paracetamol PE : mildly diffuse thyroid enlargement, markedly tenderness จงใหการ Dx 3. Organophosphate 1. Supperative thyroiditis 26. ชายอายุ 18 ป ภูมิลําเนา จ.พิจิตร 1….กอน กินซุปหนอไมกับเพื่อนอีก 3 คน ตอมาเริ่มมี 2. Acute thyroiditis อาการ weak แขนขาออนแรง BP↓ ผูปวยมารับการ intubation และใสเครื่องหายใจแลว 3. Hashimoto’s thyroiditis อาการดีขึ้น เพื่อนอีก 2 คนเสียชีวิตแลว คิดวาเกิดจากพิษใด 4. Papillary cell CA 1. Botulinum toxin 27. ผูปวยชายไทย ภูมิลําเนาสังขละ กาญจนบุรี มีรพ.ดวยอาการ proximal muscle 01Cardio weakness with hyperpigment skin, hyperkeratotic patch at palm and sole, white line nail, 31. ผูปวยมี pulse 90 irregular, PE : diastolic rumbling murmur, opening snap ขอใดถูก decreased sensation 1. มีโอกาสเกิด.....เทาคนปกติ 1. Penicillamine 2. มีโอกาสเกิด embolic stroke มากกวาคนทั่วไป 15-20 % 2. Defuroxamine 3. มีโอกาสเกิด pulmonary hypertension เพราะมีลิ้นหัวใจรั่ว 3. Calcium EDTA 32. หญิงไทยอายุ 42 ป กอนหนานี้แข็งแรงดี มีไขมา 1 wk. หอบเหนื่อยมา 3 วัน ไอ นอน 4. Hemodialysis ราบไมได T 38 C , BP 100/70 mmHg , P 100/min , RR 32/min , subconjunctival 28. ผูปวยหญิงอายุ 25 ป กินน้ํายาลางหองน้ํามา 1 ชั่วโมงกอนมาโรงพยาบาล ขอใดเปนผล hemorrhage , LV heaving (systolic murmur grade 3 at apex) มีตุมดําแดงกดเจ็บที่ปลายนิ้ว ระยะยาวที่อาจเกิดขึ้น ถาม Dx 1. Esophagitis 2. ……. pneumonitis 1. Atrial myxoma 3. Mediastinitis 4. Esophageal stricture 2. Acute rheumatoid carditis 3. Bacterial endocarditis National Test - Medicine หนาที่ 7 National Test - Medicine หนาที่ 8
  • 6. 6/41 4. Ruptured chordae tendinae 37. ผูปวยกิน amoxicillin 10 วัน อาการเจ็บคอลดลง ตอมา มีทองเสียครั้งละ 1 แกว มีมูก 33. ผูปวยไมรูสึกตัว คลําชีพจรไมได EKG : severe bradycardia with intraventricular ปนเล็กนอย 3-4 ครั้งตอวัน นาน 7 วัน เกิดจากเชื้อใด conduction delay 1. E.histolytica 2. E.coli 1. Atropine 3. Campylobactor jejuni 4. Shigella 2. Epinephrine 5. C.difficile 3. External pacing 38. ชาย 50 ป HT มานาน ปวดหลัง 1 day PTA, PE: P 110/min, BP 90/60 mmHg, 34 หญิง 55 ป CRF 2 ป, DM 2 ป, HT 4ป กินยาสม่ําเสมอ PE: BP 130/80 ตรวจตาพบ abdominal mass 8 cm with bruit. Investigation? proliferative diabetic retinopathy Lab:Cr 2 ถามปริมาณโปรตีนที่ควรกินใน 1 วัน 1. Angiogram 2. CT abdomen 1. 10 g 2. 20 g 3. MRI Abdomen 4. Acute abdomen plain film series 3. 30 g 4. 40 g 5. U/S whole abdomen 5. 50 g 39. ชาย 30 yrs, diarrhea 10 times in 8 hrs 35. เด็กหญิง 10 ขวบ เปนโรคผนังกั้นหัวใจรั่วแตกําเนิด รักษาเรื่องเหนื่อยจากหัวใจ PE : P 110/min, BP 90/60 , flat neck vein, dry lips ลมเหลวหลายครั้ง หลัง 3 ขวบ อาการดีขึ้น ครั้งนี้เหนื่อยมากมา 2 เดือน แตนอนราบได ให สารน้ําอะไร PE : central cyanosis, clubbing of fingers and toes, loud P2, diastolic blowing murmur 1. ORS 2. 5%D/W grade 3/6 at left upper sterna border, lungs-clear ครั้งนี้เหนื่อยจากอะไร 3. 5%D/N/2 4. NSS/2 1. Aortic regurgitation 2. Pulmonary embolism 5. NSS 3. Eisenmenger syndrome 40. ชายอายุ 41 ป แข็งแรงดีมาตลอด ไมมีประวัติโรคมะเร็งในครอบครัวเห็นเพื่อนรวมงาน 36. หญิงหมดสติ ให EKG มา 2 คน อายุ 41 และ 45 ปเปนมะเร็งลําไส จึงตองการตรวจเช็ค วิธีใดเหมาะสมที่สุด 1. Sinus arrest 1. Barium enema 2. Sinus bradycardia 2. Carcinoembryogenic antigen 3. Complete heart block 3. Colonoscopy 4. Left bundle branch block 4. Stool occult blood 5. Premature atrial beat 5. ไมตองทําการตรวจ screening 41. หญิง 22 ป ไมเคยเหลือง ตรวจเลือดกอนเขาทํางานพยาบาล พบ HBsAg +ve , Anti- 01 GI HBc IgM neg , LFT normal ทําอยางไร 1. ให HBV vaccine National Test - Medicine หนาที่ 9 National Test - Medicine หนาที่ 10
  • 7. 7/41 2. ให interferon alpha 2. Artesunate+tetracycline 3. ตรวจ AFP 3. Chloroquine+…. 4. U/S liver 4. Artesunate+mefloquine 5. ตรวจ HBs Ag ซ้ําอีก 6 เดือน 5. Primaquine+…. 42. ผูชายกินเหลามานาน ปวดทอง อาเจียน admit NPO แลวอาละวาด ดึงสายน้ําเกลือ เปน 47 ชาย อายุ 25 ป มาดวยเรื่องปวดศีรษะ มีไข อะไร PE :stiff neck- negative,CT brain-normal 1. Hypoglycemia CSF : protein 65 ,glucose 35,wbc??, L 95%,gram stain and india ink-negative 2. Hepatic encephalopathy 1. Acyclovir 2. Ceftriaxone 3. Wernicke encephalopathy 3. Cotrimoxazole 5. Amphotericin B 4. Delirium tremens 48. ผูปวยชายอายุ 20 ป มาดวยไขสูง หนาวสั่น ( T 38 c ) CBC: ปกติ plt100,000/ul ให 5. Alcohol intoxication Blood smear ดังรูป จงให Diagnosis 43. ผูปวยอายุ 29 ป ทองผูกบอยๆ stool exam ปกติ ทําอยางไร 1. PF 2. PV 1. GI bulk forming 3. PO 4. PM 44. ผูปวยชาย อายุ 40 ป มีกอนโตที่บริเวณคอขวามา 4 สัปดาห เดิมแข็งแรงดี ขนาดเสน 5. PV+PF ผานศูนยกลางกอน 4 ซม., firm, not fixed and no tenderness ทําอะไรตอ 49. ผูปวยชายไทยมีผื่นที่ขาหนีบ 2 ขาง ลักษณะ Annular with scale, คัน สงตรวจ KOH จะ 1. ENT exam 2. Tuberculin skin test พบลักษณะใด 3. CXR 4. Lymph node Bx 1. Branching septate hyphae 2. Short hyphae with yeast 01 ID 3. Budding yeast with thick wall 45. ผูปวย ชายไทย อายุ 30 ป มีไข 10 วัน ใหรูป blood smear มา รูปมี infected RBC ใหญ 50. ผูปวยกอนหนานี้มีไข กินยาลดไขแลวไขหายไปแลว ตอมามี erythematous nodule ที เปนอะไร ขา 2 ขาง การสงตรวจขอใด มีประโยชนในการ diagnosis 1. P. Ovale 2. P. vivax 1. CXR 3. P. Falciparum 4. P. Malariae 2. U/A 5. Mix 3. CBC 46. ผูปวยชายอายุ 30 ป มีไข ตรวจพบ double chromatin ring form ใหยาอะไร 51.ไขพยาธิดังภาพ ใหยาอะไร 1. Chloroquine 1. Albendazole National Test - Medicine หนาที่ 11 National Test - Medicine หนาที่ 12
  • 8. 8/41 2. Mebendzole 1. rickettsial infection 3. Metronidazole 2. chikungunya 4. Praziquantel 3. rheumatic heart disease 52. หญิงมาดวยอาการอาเจียนเปนเลือดสีดํา 2 ครั้ง ประวัติ Dengue ใหผล labมา ถามวาผล แทรกซอนของโรค 01 Dermato 1. Bleeding time 57. ชาย 70 ป มีผื่นนูนแดงที่ขา คัน ชอบเปนตอนปใหม Dx 2. Euglobulin test 1. Static eczema 53. ผูปวยหญิงอยูกาฬสินธุ มีอาการเหนื่อย มีไข ไอ มา 2 สัปดาห ตรวจรางกาย พบ T 2. Xerotic eczema 39°C, BP 140/60 mmHg, RR 28/min, P 100/min, Hepatomegaly 2 FB below Rt. Costal 3. Chemical eczema margin, Splenic dullness positive, CXR มี Pulmonary infiltration ที่ Lower lung field, 4. Irritant contact dermatitis ultrasound มี multiple hypoechoic lesions ที่มาม, BUN 40 mg/dl, Cr 5 mg/dl, FBS 260 5. Allergic contact dermatitis mg/dl จงใหการวินิจฉัย 1. Mellioidosis 2. Salmonella 01 Nutrition 3. Amoebic Liver Abscess 4. Leptospirosis 58. คนไขตับโต LFT ผิดปกติเล็กนอย (AST 46 U/ml, ALT 60 U/ml), U/S พบ 54. ผูปวยหญิงเปน SLE กิน prednisolone 60 mg มา 3 เดือน ไอเล็กนอย ได sputum มาเปน hepatomegaly เกิดจากการขาดอะไร ตัวอะไร 1. Zinc 2. Tretinol 1. Ancylostoma 2. Enterobius vermicularis 3. Selenium 3. Strongyloides stercolaris 4. Paragonimus 59. ผูชายกินเหลามาก มี sign of CHF เปน cardiac beri beri ถาม managemen 55. ผูปวยชายอายุ 28 ป มีไขสูงปวดศีรษะ Stiffneck +ve มีประวัติเลนน้ํา นึกถึง 1.Thiamine 2. Diazepam 1. N.fowleri 2. N.meningitis 3. glucose 3. P.aerugenosa 4. Hemophilus spp. 60. ชาย 70 ป Alzheimer กินไดนอย ไมสําลัก ตองหยอดอาหาร ทําไงดี 5. S.aureus 1. NG 2. Nasoduodenal 56. ผูปวยชาย ภูมิลําเนา จ.นราธิวาส มีไข และปวดเขา 2 ขาง ตอมามีปวดขอเทาและ 3. Nasojujunal 4. PEG ขอศอกเพิ่ม ตรวจรางกายพบวามีไข, ฟง heart no murmur, มี erythematous maculopapular 5. PEJ rash CBC มี N 40%, L 50%, atypical lymphocyte 5%, platelet ปกติ จงใหการวินิจฉัย 01 Ambu National Test - Medicine หนาที่ 13 National Test - Medicine หนาที่ 14
  • 9. 9/41 61. ผูปวยหญิง อายุ 40 ป, สูง 150 cm, น้ําหนัก 70 kg, BP 150/80 mmHg, ชอบกินอาหาร 02 Develop รสจัด จงใหการรักษา 1. เด็กซน 8 ขวบ ป.1 ซนมาก ลืมสงงาน งานหายบอย มาพบทาน.... 1. Furosemide 2. ACEI 1. เปน MR 2. พาไปพบจิตแพทย 3. B-blocker 4. CCB 3. แนะนําใหปรึกษาคุณครูที่ ร.ร. 4. เปนปกติของเด็กวัยนี้ 5. ควบคุมอาหาร + ลดน้ําหนัก 2. เด็ก 9 เดือน เกาะยืนได ยังไมเดิน พูดเปนคําไมมีความหมาย ยอมใหทุกคนอุม ฟนยังไม ขึ้น ไมยอมใหของเมื่อขอ อะไรผิดปกติ 1. walking 2. speech 3. stranger anxiety 4. tooth eruption 5. Give toys on request 3. เด็ก 2ป มาปรึกษาปญหาสุขภาพ ขณะนี้กินอาหาร 3 มื้อ ไขวันละ 1ฟอง นม4กลองตอ วัน อาหารวางเปนผลไม นน. 16kg สูง 90 cm จงใหคําแนะนํา 1. เพิ่มอาหารมื้อกอนนอน 2. เพิ่มไขมื้อละ 1ฟอง 3. ลดอาหารเปนวันละ 2มื้อ 4. ลดนมเหลือวันละ 2กลอง ตอวัน 5. เพิ่มอาหารวางเปนน้ําผลไมอีก 2มื้อ 4. เด็ก 2ป น้ําหนัก 12kg สูง 90 cm กินขาว 3มื้อ ผลไม นม 8 0z x 3, พูดได 2 คํา, วิ่งได, ได vaccine HBV x 3, OPV x 3, DPT x 3 แนะนําอยางไร 1. เพิ่มนม 2. เพิ่มมื้ออาหาร 3. รับ OPV, DPT 4. ปรึกษากุมารแพทยดานพัฒนาการ 5. ผูปวยเด็กอายุ 6 เดือน ยังชันคอไมได แขนขาขยับไดปกติดี ไมมีไข ไมซึม ผลการตรวจ รางกาย afebrile,R 30/min, BP 80/50 mmHg, P 100/min, Head circumference 46 cm, anterior fontanelle 3x3 cm, Increase muscle tone, DTR 3+, Babinski’s – dorsiflexion both sides ควรตรวจอะไรเพิ่มเติมเพื่อใหไดการวินิจฉัย 1. TORCH titer 2. Chromosome study 3. Thyroid function test 4. Bone scan National Test - Medicine หนาที่ 15 National Test - Pediatrics หนาที่ 16
  • 10. 10/41 5. U/S brain CXR : haziness 6. ผูปกครองพาเด็ก 6ขวบ มาขอใบยกเวนไมตองเขาโรงเรียน ผลการทดสอบสติปญญา จะสง Lab อะไร IQ=60 ควรทําอยางไร 1. Microplasma titer 2. ICD 1. ออกใบรับรองไมตองเขาโรงเรียนให 3. Nasopharygeal swab 2. ออกใบรับรองเขาเรียนไดโดยมีอาจารยพิเศษ 11. เด็กชาย 6 ป ไข 1 วัน เหนื่อยหอบ หายใจลําบาก PE T 40 ° , RR 33 /min, Chest : 3. ออกใบรับรองเสื่อการเขาเรียนเปน 9ป inspiratory stridor with Suprasternal retraction, inspiratory sonorous rhonchi การตรวจ 4. ไมออกใบรับรอง เพราเด็กสามารถเรียนได ในขอใดชวยวินิจฉัยมากที่สุด 5. ใหเจาหนาที่ตรวจสภาพแวดลอมบานกอนออกใบรับรอง 1.U/S chest 2. CT chest 7. เด็กอายุ 1ป แมสังเกตเห็นวาตาขวาเหลเขาในมา5เดือน พัฒนการปกติ ถาไมรักษาจะ 3. CXR PA & Lat film 4. CXR inspiration & expiration film 1. Nystagmus 2. Glaucoma 5. Lat soft tissue technique film 3. Amblyapia 4. Visual field defect 12. เด็ก 10 ป ไอ แนนหนาอก หอบ 5 วัน face, chest & upper extremities edema , liver 2 5. Limit ocular movement cm. Below Rt.Costal margin. What is the diagnosis? 1. Croup 2. Angioedema 02 Respi 3. SVC obstruction 4. CHF 8. ผูปวยเด็กชาย อายุ 6 เดือน มีไขและไอมากขึ้นหลังจากเปนหวัดมา 2 วัน, T 38°C, R 5. Constrictive pericarditis 44/min, Inspiratory stridor, retract intercostals, nasal flaring, occasionally rhonchi จงใหการวินิจฉัย 02 nephro 1.Viral croup 2.Bronchiolitis 13. เด็ก 5 ป ไข 2 วัน ปสสาวะสีเขม เหนื่อยเพลียมากขึ้น, PE mildly pale, CBC Hb 6 Hct 3.Whooping cough 4.Acute epiglottitis ... WBC normal (N85 L15) Platelet 14,300, U/A dark brown urine, urobilinogen +ve, 5.Asthmatic bronchitis Complication? 9. เด็ก 8 เดือน มีไข ซึม ไมยอมดูดนม น้ําลายยืด มา 2 วัน PE : T 39 c, RR 30 /min, 1.Gallstone 2.Acute renal failure drooling, noisy breathing, harsh breath sound, bulging posterior pharynx 3.Pulmonary embolism 1. viral croup 2. bacterial tracheitis 14. เด็กหญิง 14ป บวมทั่วตัว ขาบวม 3. retropharyngeal abscess 4. acute epiglottitis PE : T37.8 *C,BP140/90 mmHg, mild anemia 10. เด็กหญิงอายุ 8ป มีไข ไอ หอบ มา 3วัน กอนหนานี้ 4วัน มีไขต่ําๆ pitting edema both legs, oral ulcer, MP rash at palms and soles PE : Decrease breath sound at Rt. , Trachea shift to left Lab : CBC Hb 8 g/dl WBC 4,000 thrombocytopenia National Test - Pediatrics หนาที่ 17 National Test - Pediatrics หนาที่ 18
  • 11. 11/41 U/A protein 3+ WBC 30-40 cell RBC cast 19. เด็กทารกอายุ 3 วัน คลอดปกติ กินนมแมไดดี BW 3200 g ปกติดีทุกอยาง ตรวจพบวา การตรวจทางหองปฏิบัติการใดที่ชวยวินิจฉัย เหลือง TB 12 mg/dL Hct 50% ทําอยางไร 1.ASO titer 2.C3 1. ใหดูดน้ําเพิ่ม 3.... 4.ANA 2. หยุดนมแม 1-2 วัน เพราะเกิดจากการกินนมมารดา 5.albumin,cholesterol 3. ตากแดดออนๆ 15. ผูปวยเด็ก อายุ 6 ป อาเจียนมาก ตรวจ Electrolyte ผลที่ไดจะเปนอยางไร 4. กลับบานไมไดใหobserve 1,Hypochloremia 2.Hypokalemia 5. admit on photo 3.Metabolic alkalosis 20. ผูปวยเด็ก preterm BW = 2,200 g มารดาคลอด C/S due to abruptio placenta apgar 16. เด็กชาย อายุ 6 สัปดาหแมพามาเพราะอาเจียนมาก serum E’lyte เขาไดกับ score 5, 8 1. hyperchloremic hyponatremic metabolic acidosis หลังคลอด 12 ชม. เหลือง ทํา exchange สาเหตุสําคัญที่ตองตรวจการไดยินของเด็กคนนี้ 2. hyperchloremic hypokalemic metabolic acidosis คือ 3. hyperchloremic hyperkalemic metabolic acidosis 1. BW = 2,200 g 2. abruptio placenta 4. hypochloremic hypokalemic metabolic alkalosis 3. apgar score 5, 8 4. preterm 5. hypochloremic hyponatremic metabolic alkalosis 5.เหลือง 17. เด็ก 8 ป bilateral vesico ureteral reflux มีติดเชื้อซ้ําบอยๆ BUN/Cr ขึ้นเรื่อยๆ ลาสุด 21. ทารกอายุครรภ 44 สัปดาห น้ําหนักแรกคลอด 3400 g มี meconium stained BUN 55, Cr 3.8, Na 135, K 4.3, HCO3 16, ปสสาวะตอวัน 900 ความดันปกติ จงใหการ amniotic fluid คลอดทางชองคลอด APGAR score 2,5 ที่ 1 และ 5 นาที ตามลําดับ รักษา หลังคลอด 10 ชม. มีชักเกร็งกระตุกทั่วตัว ผลตรวจเลือด blood glucose 50 mg/dl Hct 1. salt diet ลดลง 2. K diet ลดลง 58% สาเหตุการชักในทารกรายนี้จากความผิดปกติใด 3. oral NaHCO3 4. phosphate diet 1. meningitis 2. hypoglycemia 5. dialysis 3. polycythemia 4. perinatal asphyxia 5. intraventricular hemorrhage 02 Neonate 22. เด็กคลอดกอนกําหนด แมเปน DM หายใจหอบเหนื่อย เร็ว CXR พบ hyperaeration 18. เด็กแรกเกิดเหลืองหลังคลอด 3-4 wk กินนมแม ไมโต ลิ้นจุกปาก ไมดูดนม ทองอืด จะ with generalized haziness with air bonchogram, Dx? ตรวจอะไรเพื่อ Dx 1. fluid 2. MAS 1. Chromosome study 2. TFT 3. RDS 4. bronchopulmonary dysplasia 3. GH level 4. GI follow through 5. transient tachypnea of the newborn National Test - Pediatrics หนาที่ 19 National Test - Pediatrics หนาที่ 20
  • 12. 12/41 23. เด็กปากแหวง เพดานโหว นิ้วเกิน คิดถึง syndrome ? 02 ID 1. Down’s 2. Patau’s 27. เด็กอายุ 18 เดือน มีปูเปน TB รักษากิน anti-TB มา 2 สัปดาห เด็กไมมีอาการ ตรวจ 3. Turner’s 4. Edward’s รางกายปกติ CXR ปกติ PDD 15 mm. ใหการรักษาที่เหมาะสม 5. Klinefelter’s 1. INH 9 mo 2. INH + rifampin 6 mo 24. หญิงอายุ 23 ป G1 GA 42 weeks มีอาการเจ็บครรภ น้ําเดินมา 2 ชั่วโมง พบวามีขี้เทาปน 28. เด็ก 2 ป มาคลินิกเด็กสบาย ได vaccine MMR*1, HBV*3, OPV*3, DPT*3, JE*2 ครั้ง ในน้ําคร่ํา มี fetal distress FHR 80 /min แพทยทํา emergency C/S ออกมาพบไดทารก BW นี้จะใหอะไร 3,000 g Apgar score 2, 3 ที่ 1 และ 5 นาที ตอมามีเขียว หายใจลําบาก และซีด ถามวาพบ 1. OPV, DTP 2. Hib , JE ความผิดปกติใดไดในทารกรายนี้ 3. OPV, DPT, JE 4. OPV, DPT, Hib 1. anemia 2. thrombocytosis 5. OPV, DPT, JE, Hib 3. hyperactive bowel sound 4. hypotension 29. เกี่ยวกับ roseola infantum ขอใดผิด 5. pulmonary arterial hypertension 1. ผื่นขึ้นวันที่ 3 2. เกิดจาก HSV 16 25. เด็กแรกเกิด หนัก ... (รูสึกวาจะหนักปกติ) แมคลอด no complication แรกเกิด มี Hct 3. ผื่นขึ้นที่ลําตัวกอน 68%, glucose 55 mg/dl จะ management อยางไร 30. เด็กเลนน้ํา มี stiff neck มีไข เกิดจากเชื้อใด 1. early feeding 2. 5% D N/2 6 mg/kg/day IV 1. Naegleria fowleri 3. 10% D N/2 2 mg IV push ชาๆ 4. Hct ซ้ําในอีก 6 ชั่วโมง 31. เด็ก 7 ป ~ 3 วันกอน อาเจียนมาก ไมแนใจวามีไขหรือเปลา, 1 วันกอน อาเจียนเปน 5. exchange transfusion เลือด 2 ครั้ง ตรวจรางกาย BP ต่ําๆ , pulse เร็วเล็กนอย , CBC ซีด ,WBC ~2,000 , 26. ด็กชาย Term 3,000 g normal labour 26hr หลังคลอด แมเลือด Group O Rh +ve ลูก Lymphocyte เดน, Plt. ต่ํา, prolong PT /PTT , fibrinogen 120 (200-400) ควรตรวจอะไร Group A Rh +ve CBC Hct 40% WBC 11,000,Platelet 260,000 Peripheral blood smear เพื่อวินิจฉัยภาวะแทรกซอนในรายนี้ microspherocyte 2+, reticulocyte 10%, Coomb’s test +ve 1.D-dimer 2.Bleeding time ทําอยางไร 3.Euglobulin lysis time 4.Clot generation time 1. single photo Tx 32. เด็กอายุ 1 ป มีไขสูง 39.5°C ความดันโลหิต 80/60 mmHg, P 140/min มี petechiae ที่ 2. double photo Tx แขนและขา 3. Intensive photo Tx จะ Management อยางไร 4. partial plasma transfusion 1.Dextran 2.Dopamine 5. total plasma transfusion 3.Platelet 4.NSS 5.FFP National Test - Pediatrics หนาที่ 21 National Test - Pediatrics หนาที่ 22
  • 13. 13/41 33. เด็กหญิง มีไข ปวดหูขวา ได amoxy syr 2 day ไขไมลด ปวดหูมากขึ้น การปฏิบัติที่เห 37. ผูปวยหญิงอายุ 16 ป เมื่อ 4 สัปดาหกอนเปนไข เจ็บคอ ซื้อยากินเอง 2 วัน อาการดีขึ้น มะสมตอไปคือ ตอมามีปวดเขา 2 ขาง ปวดขอเทาซาย T 38°, no pharyngeal injection, swelling and 1. oral cefaclor 2. oral amoxy/clav tenderness out both knee and Lt.ankle, pericardial mb, subcutaneous nodule at 3. oral erythromycin 4. iv ampicillin Lt.forearm จงใหการวินิจฉัย 5. tympanosynthesis 1. Acute rheumatic fever 2. Gonococcal arthritis 3. SLE 02 Nutrition 38. เด็กชายอายุ 5 ป แข็งแรงดี มาฉีดวัคซีนตามเกณฑ รางกายแข็งแรง PE no cyanosis, 34. แมพาบุตรอายุ 7 เดือน มาตรวจ ชันคอไดแตยังไมคว่ํา กระหมอมหนายังไมปด มีเด็ก normal S1, widely fixed split S2 grade II/VI systolic murmur at left upper sternal border ในหมูบานเปนหลายคน มาตรการปองกันคือ diagnosis? 1. เสริม iodine ใหหญิงตั้งครรภ ทุกคนในหมูบาน 1. innocent murmur 2. pulmonary stenosis 2. ให thyroid hormone แกเด็กแรกเกิดทุกคน 3. aortic stenosis 4. ASD 3. สงเสริมใหกินเกลือผสม iodine ทั้งหมูบาน 5. VSD 4. เสริมเกลือ iodine ในโรงเรียน 39. เด็กอายุ10ป มีประวัติเปนผนังกั้นหองหัวใจพิการแตกําเนิดมีอาการเขียว และเคยหอบ 5. ให thyroid hormone แกหญิงตั้งครรภทุกคน เหนื่อยตั้งแตอายุ 2-3ป ตรวจรางกายพบ diasteric blowing murmur ,clubbing finger ถามวา 02 GI อาการเขียวเกิดจากอะไร 35. เด็กอายุ 2 ป 6kg ยาว 55 cm AF 3x4 cm PF ยังไมปด ยังชันคอไมได ขยับแขนขาได 1. Eisenmewnger 2. Hypoxic spell ซีด ทองผูก ถาย 2-3 วัน/ครั้ง อึเปนเม็ดกระสุน 40. เด็กทารกอายุ 2 วัน คลอดที่บาน มาดวยเหนื่อย เขียว 1. Hypothyroidism 2. Functional constipation PE : พบ floppy, cyanosis, HR 50 /min, RR 100 /min ควรทําอะไรเปนอันดับแรก 3. Hirschsprung‘s disease 4.Iron def anemia 1. Glucose 4. Adrenaline 36. เด็ก NL apgar 9,10 แมฝากครรภปกติ Suction ไดเลือด 5 cc สงตรวจอะไร 2. NSS 5. Chest compression 1. Apt test 3. PPV 2. Coag 3. CBC 02 Critical care 41. เด็ก 8 ป DKA BP 90/70 poor perfusion Na 145, K 3, Cl 110, HCO3 10, urine ketone 02 Cardio 4+ ให fluid ใน 1 hr. แรกเทาไร 1. 0.9 % NaCl 20 ml/kg 2. 0.45 % NaCl 20 ml/kg National Test - Pediatrics หนาที่ 23 National Test - Pediatrics หนาที่ 24
  • 14. 14/41 3. RLS 20 ml/kg 4. 0.9 % NSS + KCl 20 mEq/kg 1. ขอความยินยอมพอแมเพื่อ LP 5. 0.45 % NSS + KCl 20 mEq/kg 2. LP เลยเพราะเรงดวน 42. เด็กอายุ 2ป หายใจลําบาก เปนทันทีขณะเลนอยูกับพี่ชาย PE: T 37c RR40/min PR 3. CT brain 140/min 4. ใหยากันชัก, ATB หลัง H/C Lung : fair air entry, Inspiratory+expiratory stridor, suprasternal+subcostal retraction 5. สงตอ ถาม Mx 46. เด็กหญิงอายุ...ป มีอาการเหมอลอย พัฒนาการปกติ สติปญญาปกติ ตรวจคลื่นไฟฟา 1. Back blow-chest thrust 2. Abdominal thrust สมองพบ 3 Hz wave ใน hyperventilation การรักษาที่เหมาะสมที่สุดในผูปวยรายนี้คือ 3.Heimleich maneuver 4. ET tube 1. ACTH 2. Phenytoin 5. Bronchoscopy 3. Vigabatrin 4. Valproic acid 43. เด็กชาย 1 ป ไขสูง ซึม มา 1 วัน ตรวจรางกายพบ T 39.2 C, BP 80/60 mmHg, RR 42 5. Phenobarbital /min, P 140 bpm มี petichiae + purpura at both lower extremities อะไรที่ควรให 47. เด็ก ชักมานาน 30 นาที ไมรูสึกตัว ที่ ER ยังชักอยู เปน generalized tonic clonic seizure 1. Dextran ควรใหยาใด 2. Doputamine 1. Diazepam 2. Phenytoin 3. Platelet conc. 3. Valproic acid 4. Phenobarbital 4. FFP 5. Carbamazepine 5. NSS 02 Hemato 02 Neuro 48. ด.ช. 3 ป มีไขมา 1 วัน 44. เด็กชาย 5 เดือน มีอาการผงกศีรษะและเกร็งแขนขาทั้งสองขางเปนชุดๆ ชุดละ 2-3 นาที PE: mildly pale, moderate icteric sclera, liver 2 cm below RCM, spleen just palpable มักเปนตอนเชาหลังตื่นนอน no eye contact, poor neck control, Spastic tone of all CBC: Hb 7 Hct 22 wbc 45,000 (N 80 L 40??) platelet ปกติ extremities, DTR 1+ all จะใหการรักษาอยางไร PBS: microcytic 2+, hypochromic 2+, anisocytosis 2+, poikilocytosis 2+, schistocytosis 1. diazepam 2. phenytoin 1+, few ovalocyte and target cell 3. Phenobarbital 4. สงกระตุนพัฒนาการ จะสงตรวจ Investigation อะไรเพื่อชวยในการวินิจฉัย 5. สงตอกุมารแพทย neuro 1. D-dimer 2. G6PD level 45. เด็กอายุ 8 เดือน แมพามาดวยอาการไขสูง , anterior fontanel โปง , Brudzinski 3. serum ferritin 4. Hb typing positive จะทําอยางไร National Test - Pediatrics หนาที่ 25 National Test - Pediatrics หนาที่ 26
  • 15. 15/41 49. เด็กอายุ 9 เดือน มารดาเลี้ยงดวยนมขนตั้งแต 4 เดือน แมพามาฉีดวัคซีน มีซีดเล็กนอย 1. NG lavage 2. Atropine MCV 67,microcytic 1+, anisocytosis 2+, WBC& Plt. ปกติ สาเหตุเกิดจากอะไร 3. Naloxone 4. Activated charcoal 1. Thalassemia Hb H disease 2. Iron deficency 3. Folate deficiency 4. G-6PD deficiency 02 Immuno 5. Hb E 55. เด็ก 10 ป เปนโรคหืดมา 5 ป รักษาไมสม่ําเสมอ มีอาการเดือนละหลายครั้ง ครั้งนี้ 50. เด็ก 4 ขวบ ซึม ant fontanelle bulging ชัก กินแตนมขนหวาน เหนื่อย หายใจลําบาก กินยาขยายหลอดลมไป 1 เม็ดเมื่อ1 ชม.กอน อาการไมดีขึ้น ที่รพ. 1. Arginine prothrombin complex deficiency ไดพนยา Salbutamol ไป 2 doses ยังมีอาการหอบเหนื่อยอยู ควรทําอยางไรตอไป 51. เด็กชายอายุ 1 ป ไขมา 2 วัน 1. NB corticosteroid 2. IV corticosteroid PE : moderately pallor, mild jaundice, tachycardia, no hepatosplenomegaly 3. NB theophylline 4. NB ipratropium CBC: Hct 24% WBC 115,000(N 76% L21%) Plt. 470,000 56. เด็ก 3 เดือน disseminater BCG infection อยูรพ. ติด varicella PE: vesicle, pneumonia ให bloodsmear มาพบ contracted Hb น้ําหนักนอย ภาวะภูมิคุมกันผิดปกติจากอะไร Diagnosis? 1. T-cell 2. Phagocytosis 1. Hb H disease 2. G6PD deficency 3. Opsonization 4. Ig 3. hereditary spherocytosis 4. AIHA 5. complement 52. เด็กชาย 5 ป มีจ้ําเขียวตามแขนขา PE อื่นๆ ปกติ CBC E13% Plt. 180,000 Dx 1. Hemophillia 2. vWD 02 Ambulatory 3. Hypereosinophil 4. ITP 57. เด็ก 8 เดือน มี labia minora ติดกัน เห็น clitoris ปกติ ไมเห็น hymen/vagina ปสสาวะ 5. APDE อุจจาระ ปกติ 53. เด็กหญิง ซีด เหลืองเล็กนอย ตับไมโต มามโต MCV 62 ถาม Dx จะทําการรักษาอยางไร 1. Thalassemia 2. G-6-PD deficency 1. ทา estrogen ที่ labia minora 3. Iron deficiency 4. AIHA 2. ทา ATB ที่ labia minora 5. HS 3. ผาตัดเปด 02 Toxic 4. รักษาความสะอาด แลว ติดตามตอน 2 ป 54. เด็กอายุ 5 ป กินสารไมทราบชนิด มี lethargy, T 37 c, BP 80/50 mmHg, RR 20 /min, 5. ไมตองรักษา คอยมาหาถามีปญหาประจําเดือนไมมา PR 130 /min, secretion from mouth, pupil 1 mm, มี sweating ควรใหการ management อยางไร National Test - Pediatrics หนาที่ 27 National Test - Pediatrics หนาที่ 28
  • 16. 16/41 1. ตรวจ thalassemia 03 OB 6. ผูหญิง G2 GA 25 wk มี Hx ลูกคนแรกแทงจากการบวมน้ํา จะตรวจวาลูกในทอง มีภาวะ 1. หญิงไทย 28 yr G1P1 ใหนมลูก 6 wk ตองการคุมกําเนิด 2-3 yr มักมีอาการปวดทองนอย บวมน้ําไดอยางไร เวลามีประจําเดือน ควรใชยาคุมอะไร 1. amniocentesis 3. ตรวจเลือดแมจาก fetal cells 1. Combinded pill 2. chordocentesis 4. Ultrasound 2. Pregestin only pill 7. monitor ได variable deceleration ถาม Mx 3. DMPA 1. induction of labor 2. augmentation of labor 2. คลํา sacrumpromontery ไมได ischial spine หางกัน 11 cm. Interbulous หางกัน 8 cm. 3. c/s ประมาณวาเวลาผานไป มดลูกหดตัวดี แตปากมดลูกเปดเทาเดิม มีความผิดปกติที่ใด 8. GA 20 weeks มีกลิ่นและคันที่ชองคลอดมา 2 สัปดาห ตรวจรางกายพบวา แดงที่ปาก 1. midplane ชองคลอด ตกขาวสีเหลืองเขียว มีฟอง ชองคลอดแดง cervix สีแดงคล้ํา (ไมบอกวามี curd) 2. outlet จะใหยาอะไรรักษา 3. inlet 1. oral doxycycline 2. oral tinidazole 3. หญิงไทยอายุ 30 ป G1P0 GA 8 wk มาดวยเลือดออกกะปริดกะปรอยทางชองคลอดมา 3. oral ketoconazole 4. oral fluconazole 2 วัน มีใจสั่น คลื่นไสอาเจียนมาก ไมมีอาการปวดทอง ตรวจรางกายพบ not pale , uterus 5. clotrimazole Vg at pubic symphysis, BP 140/90, P 110/min, RR 20/min, urine sugar-neg , urine protein- 9. ผูปวยหญิง GA 41 wk ครรภที่ 4 มาดวย labour pain ได drip oxytocin แลว uterine neg จงใหการวินิจฉัยที่เปนไปไดมากที่สุด contraction นาน 4 hr 35 min ตอมาเจ็บทองมากขึ้น มดลูกแข็งตัว คอดตรงกลาง FHR 140 1. Twins 4. Trophoblastic disease m/ จงใหการวินิจฉัย 2. Thyrotoxicosis 5. Pregnancy with myoma uteri 1. Hyperstimulation uterus 2. Treaten rupture uterus 3. preeclampsia 10. หญิงอายุ 34 ป GA 34 wk มาดวยมีเลือดออกทางง vagina เปนเลือดปนกอนเลือด 4. หญิงไทย G2 มาฝากครรภ GA 18 wk ตรวจเลือดแม A- พอ B+ (อาจสลับกัน ประมาณ painless, no uterine contraction , เด็กไมดิ้น จงใหการ management นี้) ถามทําไรตอ 1. CBC 2. Coagulogram 1. Anti D 4. Cordocentesis 3. Ultrasound 4. Non stress test 2. direct comb 5. Amniocentesis 5. Per vaginal examination with sterile speculum 3. indirect coomb 03 Gyne 5. ผูปวยหญิงมาฝากครรภครั้งแรก ตรวจ osmotic fragility test positive ตองตรวจอะไร 11. การใชยา emergency contraception เปนอันดับแรก 1.ขนาด 60 2 เม็ด พรอมกัน National Test - Obstetrics and Gynecology หนาที่ 29 National Test - Obstetrics and Gynecology หนาที่ 30
  • 17. 17/41 2.ขนาด 100 2 เม็ดหางกัน 12 ชม. 2. Endometrial Bx 3.ขนาด 60 2 เม็ดหางกัน 12 ชม. 3. Fractional and curettage 4.ขนาด 100 และ 60 2 เม็ดหางกัน 12 ชม. 12. หญิง painless ulcer 1 cm ที่ปากชองคลอด elevated border, yellow vaginal discharge ถาม Ix 1. VDRL 4. G/S 2. Culture 5. Biopsy 3. AFB 13. หญิงอายุ 34 ป GA 36wk มีอาการปสสาวะแสบขัด และเจ็บที่บริเวณชองคลอด ตรวจ รางกายพบ vesicular lesion with shallow ulcer at vulva จงใหการวินิจฉัย 1. Herpetic vulva 3. Candidiasis at vulva 2.Molluscum contagiosum 4. Syphillis 14. หญิงอายุ 35 ป น้ําหนัก 70 kg สูง 155 cm มีประจําเดือน 2-3 เดือนตอครั้ง ครั้งนี้ ประจําเดือนขาดไป 2 เดือน ตอมามีเลือดออกปริมาณมาก นาน 10 วัน ปฏิเสธประวัติ เพศสัมพันธ PE ปกติ สาเหตุของประจําเดือนผิดปกติในรายนี้คือ 1. anovulation 4. coagulation defect 2. irregulartory shedding 5. corpus luteal insufficiency 3. irregulation ripening 15. หญิงอายุ 27 ประจําเดือนคลาดเคลื่อนมา 3 wk มีเลือดออกกระปดกระปรอยมา 4 วัน มดลูกโตเล็กนอย PV: พบเลือด os close ไมพบกอนที่ adnexa UPT post. สงอะไร DX 1. Pap-smear 4. Transvaginal U/S 2. AFP 5. Transabd U/S 3. Serum Beta-HCG 16. ญ หมดประจําเดือนแลว มีเลือดออกทางชองคลอดประมาณ 2 wks U/S พบ endometrium หนา 2 mm ควรทําอยางไร 1. Hysterectomy National Test - Obstetrics and Gynecology หนาที่ 31 National Test - Obstetrics and Gynecology หนาที่ 32
  • 18. 18/41 04 GenSx 6. ผูปวยหญิงไทย อายุ 56 ป ปวดทองดานบนขวา 2 วัน มีไข ไมคลื่นไสอาเจียน มีประวัติ 1. ผูปวยชายมีไข หนาวสั่น ปวดทอง RLQ กดเจ็บ U/S พบ CBD dilate, multiple ปวดเปนๆหายๆ มา 2 ป ตรวจรางกาย : T 39, HR…, RR…, Abdomen: soft tenderness at hypoechoic mass at segment 6 RUQ investigations : TB 5.8, DB 4.2, SGOT 9, SGPT 120, DLP 168 จะใหยาอะไร 1.Hepatoma 3.Amebic liver abscess 1. Ciprofloxacin 2.CholangioCA 4.pyogenic liver abscess 2. Ceftriaxone + … 5. Infected Cyst 3. Ceftriaxone + Metronidazole 2. เด็กชาย อายุ 5 ปปวดทองมาก ทองเสีย 2 ครั้ง อาเจียน 3 ครั้ง ปวดรอบ umbulicus+RUQ 4. Ampicilin + Gentamycin สง investigate อะไร 5. Gentamycin + Metronidazole 1.lipase 4.CBC 7. ผูปวยชายอายุ 40 ป มีประวัติกอนเขาๆ ออกๆ ที่อัณฑะ ตอนปวด บวม แดง กอนออกมา 2.amylase 5.U/A แลว จงใหการวินิจฉัย 3.film acute abdomen series 1. orchitis 2. torsion testis 3. ชาย 55 ป ดื่มสุรา cirrhosis มา7 ป, U/S -liver:solid, ill-defined mass 7 cm 3. hydrocelitis 4. incarcerated hernia Investigate? 8. ผูปวยหญิงอายุ 52 ป เปน cholecystitis ไมไดรับการรักษา 4 วันกอนมีอาการปวดทอง 1.CEA 4.AFP บริเวณลิ้นป จะใหนอนแนนมากขึ้น มีปวดราวไปหลัง อาเจียนเปนอาหาร ตรวจรางกายพบ 2.CA19-9 5.ALP v/s stable มี guarding and rebound tenderness at upper abdomen จงใหการวินิจฉัย 3.Beta-hCG 1. acute cholecystitis 2. acute appendicitis 4. หญิงไทย 25 ป กินน้ํายาลางหองน้ํามา ผลแทรกซอนระยะยาว 3. peptic perforate 4. acute pancreatitis 1. esophagitis 4. chemical pneumonitis 5. small bowel obstruction 2 mucositis 5. stricture esophagus 3. mediastinitis 04 HNB 9. เด็กหญิงอายุ 9 ป มารดาพามาดวยเรื่องกอนที่เตานม กดเจ็บ P.E. พบ mass เสนผาน ศูนยกลาง 1.5 cm under areolar, tender lump, no lymphadenopathy อื่นๆปกติ จงDx 5. ผูปวยหญิงทองผูก+ปวดทอง RLQ film เจอ …………………ถามวา management คือ 1. breast bud 4. gynecomastia 1. CT abdomen 2. fibroadenoma 5. breast abscess 2. Diverticulectomy 3. fat lump 3. F/U Barium enema ทุกป National Test - Surgery หนาที่ 33 National Test - Surgery หนาที่ 34